Как найти базис векторного пространства

В статье о n-мерных векторах мы пришли к понятию линейного пространства, порождаемого множеством n-мерных векторов. Теперь нам предстоит рассмотреть не менее важные понятия, такие как размерность и базис векторного пространства. Они напрямую связаны с понятием линейно независимой системы векторов, так что дополнительно рекомендуется напомнить себе основы и этой темы.

Введем некоторые определения.

Определение 1

Размерность векторного пространства – число, соответствующее максимальному количеству линейно независимых векторов в этом пространстве.

Определение 2

Базис векторного пространства – совокупность линейно независимых векторов, упорядоченная и в своей численности равная размерности пространства.

Рассмотрим некое пространство n-векторов. Размерность его соответственно равна n. Возьмем систему из n-единичных векторов:

e(1)=(1, 0,…,0)e(2)=(0, 1,…,0)e(n)=(0, 0,…,1)

Используем эти векторы в качестве составляющих матрицы A: она будет являться единичной с размерностью n на n. Ранг этой матрицы равен n. Следовательно, векторная система e(1), e(2),…, e(n) является линейно независимой. При этом к системе невозможно добавить ни одного вектора, не нарушив ее линейной независимости.

Так как число векторов в системе равно n, то размерность пространства n-мерных векторов равна n, а единичные векторы e(1), e(2),…, e(n) являются базисом указанного пространства.

Из полученного определения сделаем вывод: любая система n-мерных векторов, в которой число векторов меньше n, не является базисом пространства.

Если мы поменяем местами первый и второй вектор, получим систему векторов e(2), e(1),…, e(n). Она также будет являться базисом n-мерного векторного пространства. Составим матрицу, взяв за ее строки векторы полученной системы. Матрица может быть получена из единичной матрицы перестановкой местами первых двух строк, ранг ее будет равен n. Система e(2), e(1),…, e(n) линейно независима и является базисом n-мерного векторного пространства.

Переставив местами в исходной системе другие векторы, получим еще один базис.

Мы можем взять линейно независимую систему неединичных векторов, и она также будет представлять собой базис n-мерного векторного пространства.

Определение 3

Векторное пространство с размерностью n имеет столько базисов, сколько существует линейно независимых систем из n-мерных векторов числом n.

Плоскость является двумерным пространством – ее базисом будут два любых неколлинеарных вектора. Базисом трехмерного пространства послужат три любых некомпланарных вектора.

Рассмотрим применение данной теории на конкретных примерах.

Пример 1

Исходные данные: векторы

a=(3, -2, 1)b=(2, 1, 2)c=(3, -1, -2)

Необходимо определить, являются ли указанные векторы базисом трехмерного векторного пространства.

Решение

Для решения поставленной задачи исследуем заданную систему векторов на линейную зависимость. Составим матрицу, где строки – координаты векторов. Определим ранг матрицы.

A=323-21-112-2A=3-212123-1-2=3·1·(-2)+(-2)·2·3+1·2·(-1)-1·1·3-(-2)·2·(-2)-3·2·(-1)==-25≠0⇒Rank(A)=3

Следовательно, заданные условием задачи векторы линейно независимы, и их численность равна размерности векторного пространства – они являются базисом векторного пространства.

Ответ: указанные векторы являются базисом векторного пространства.

Пример 2

Исходные данные: векторы

a=(3, -2, 1)b=(2, 1, 2)c=(3, -1, -2)d=(0, 1, 2)

Необходимо определить, может ли указанная система векторов являться базисом трехмерного пространства.

Решение

Указанная в условии задачи система векторов является линейно зависимой, т.к. максимальное число линейно независимых векторов равно 3. Таким образом, указанная система векторов не может служить базисом трехмерного векторного пространства. Но стоит отметить, что подсистема исходной системы a=(3, -2, 1), b=(2, 1, 2), c=(3, -1, -2) является базисом.

Ответ: указанная система векторов не является базисом.

Пример 3

Исходные данные: векторы

a=(1, 2, 3, 3)b=(2, 5, 6, 8)c=(1, 3, 2, 4)d=(2, 5, 4, 7)

Могут ли они являться базисом четырехмерного пространства?

Решение

Cоставим матрицу, используя в качестве строк координаты заданных векторов

A=1233256813242547

По методу Гаусса определим ранг матрицы:

A=1233256813242547~1233010201-1101-21~~1233010200-1-100-2-1~1233010200-1-10001⇒⇒Rank(A)=4

Следовательно, система заданных векторов линейно независима и их численность равна размерности векторного пространства – они являются базисом четырехмерного векторного пространства.

Ответ: заданные векторы являются базисом четырехмерного пространства.

Пример 4

Исходные данные: векторы

a(1)=(1, 2, -1, -2)a(2)=(0, 2, 1, -3)a(3)=(1, 0, 0, 5)

Составляют ли они базис пространства размерностью 4?

Решение

Исходная система векторов линейно независима, но численность векторов в ней недостаточна, чтобы стать базисом четырехмерного пространства.

Ответ: нет, не составляют.

Разложение вектора по базису

Примем, что произвольные векторы e(1), e(2),…, e(n) являются базисом векторного n-мерного пространства. Добавим к ним некий n-мерный вектор x→: полученная система векторов станет линейно зависимой. Свойства линейной зависимости гласят, что хотя бы один из векторов такой системы может линейно выражаться через остальные. Переформулируя это утверждение, можно говорить о том, что хотя бы один из векторов линейно зависимой системы может раскладываться по остальным векторам.

Таким образом, мы пришли к формулировке важнейшей теоремы:

Определение 4

Любой вектор n-мерного векторного пространства единственным образом раскладывается по базису.

Доказательство 1

Докажем эту теорему:

зададим базис n-мерного векторного пространства – e(1), e(2),…, e(n). Сделаем систему линейно зависимой, добавив к ней n-мерный вектор x→. Этот вектор может быть линейно выражен через исходные векторы e:

x=x1·e(1)+x2·e(2)+…+xn·e(n) , где x1, x2,…, xn – некоторые числа.

Теперь докажем, что такое разложение является единственным. Предположим, что это не так и существует еще одно подобное разложение:

x=x~1e(1)+x2~e(2)+…+x~ne(n), где x~1, x~2,…, x~n – некие числа.

Отнимем от левой и правой частей этого равенства соответственно левую и правую части равенства x=x1·e(1)+x2·e(2)+…+xn·e(n) . Получим:

0=(x~1-x1)·e(1)+(x~2-x2)·e(2)+…(x~n-xn)·e(2)

Система базисных векторов e(1), e(2),…, e(n) линейно независима; по определению линейной независимости системы векторов равенство выше возможно только тогда, когда все коэффициенты (x~1-x1), (x~2-x2),…, (x~n-xn) будут равны нулю. Из чего справедливым будет: x1=x~1, x2=x~2,…, xn=x~n. И это доказывает единственный вариант разложения вектора по базису.

При этом коэффициенты x1, x2,…, xn называются координатами вектора x→ в базисе e(1), e(2),…, e(n).

Доказанная теория делает понятным выражение «задан n-мерный вектор x=(x1, x2,…, xn)»: рассматривается вектор x→ n-мерного векторного пространства, и его координаты заданы в некотором базисе. При этом также понятно, что этот же вектор в другом базисе n-мерного пространства будет иметь другие координаты.

Рассмотрим следующий пример: допустим, что в некотором базисе n-мерного векторного пространства задана система из n линейно независимых векторов

e(1)=(e1(1), e2(1),…, en(1))e(2)=(e1(2), e2(2),…, en(2))⋮e(n)=(e1(n), e2(n),…, en(n))

а также задан вектор x=(x1, x2,…, xn).

Векторы e1(1), e2(2),…, en(n) в этом случае также являются базисом этого векторного пространства.

Предположим, что необходимо определить координаты вектора x→ в базисе e1(1), e2(2),…, en(n), обозначаемые как x~1, x~2,…, x~n.

Вектор x→ будет представлен следующим образом:

x=x~1·e(1)+x~2·e(2)+…+x~n·e(n)

Запишем это выражение в координатной форме:

(x1, x2,…, xn)=x~1·(e(1)1, e(1)2,…, e(1)n)+x~2·(e(2)1, e(2)2,…, e(2)n)+…++x~n·(e(n)1, e(n)2,…, e(n)n)==(x~1e1(1)+x~2e1(2)+…+x~ne1(n), x~1e2(1)+x~2e2(2)++…+x~ne2(n), …, x~1en(1)+x~2en(2)+…+x~nen(n))

Полученное равенство равносильно системе из n линейных алгебраических выражений с n неизвестными линейными переменными x~1, x~2,…, x~n:

x1=x~1e11+x~2e12+…+x~ne1nx2=x~1e21+x~2e22+…+x~ne2n⋮xn=x~1en1+x~2en2+…+x~nenn

Матрица этой системы будет иметь следующий вид:

e1(1)e1(2)⋯e1(n)e2(1)e2(2)⋯e2(n)⋮⋮⋮⋮en(1)en(2)⋯en(n)

Пусть это будет матрица A, и ее столбцы – векторы линейно независимой системы векторов e1(1), e2(2),…, en(n). Ранг матрицы – n, и ее определитель отличен от нуля. Это свидетельствует о том, что система уравнений имеет единственное решение, определяемое любым удобным способом: к примеру, методом Крамера или матричным методом. Таким образом мы сможем определить координаты x~1, x~2,…, x~n вектора x→ в базисе e1(1), e2(2),…, en(n).

Применим рассмотренную теорию на конкретном примере.

Пример 6

Исходные данные: в базисе трехмерного пространства заданы векторы

e(1)=(1,-1,1)e(2)=(3, 2, -5)e(3)=(2, 1, -3)x=(6, 2, -7)

Необходимо подтвердить факт, что система векторов e(1), e(2), e(3) также служит базисом заданного пространства, а также определить координаты вектора х в заданном базисе.

Решение

Система векторов e(1), e(2), e(3) будет являться базисом трехмерного пространства, если она линейно независима. Выясним эту возможность, определив ранг матрицы A, строки которой – заданные векторы e(1), e(2), e(3).

Используем метод Гаусса:

A=1-1132-521-3~1-1105-803-5~1-1105-800-15

Rank (A) = 3. Таким образом, система векторов e(1), e(2), e(3) линейно независима и является базисом.

Пусть в базисе вектор x→ имеет координаты x~1, x~2, x~3. Связь этих координат определяется уравнением:

x1=x~1e1(1)+x~2e1(2)+x~3e1(3)x2=x~1e2(1)+x~2e2(2)+x~3e2(3)x3=x~1e3(1)+x~2e3(2)+x~3e3(3)

Применим значения согласно условиям задачи:

x~1+3x~2+2x~3=6-x~1+2x~2+x~3=2x~1-5x~2-3×3=-7

Решим систему уравнений методом Крамера:

∆=132-1211-5-3=-1∆x~1=632221-7-5-3=-1,     x~1=∆x~1∆=-1-1=1∆x~2=162-1211-7-3=-1,     x~2=∆x~2∆=-1-1=1∆x~3=136-1221-5-7=-1,     x~3=∆x~3∆=-1-1=1

Так, вектор x→ в базисе e(1), e(2), e(3) имеет координаты x~1=1, x~2=1, x~3=1.

Ответ: x=(1,1,1)

Связь между базисами

Предположим, что в некотором базисе n-мерного векторного пространства даны две линейно независимые системы векторов:

c(1)=(c1(1), c2(1),…, cn(1))c(2)=(c1(2), c2(2),…, cn(2))⋮c(n)=(c1(n), e2(n),…, cn(n))

И

e(1)=(e1(1), e2(1),…, en(1))e(2)=(e1(2), e2(2),…, en(2))⋮e(n)=(e1(n), e2(n),…, en(n))

Указанные системы являются также базисами заданного пространства.

Пусть c~1(1), c~2(1),…, c~n(1) – координаты вектора c(1) в базисе e(1), e(2),…, e(3), тогда связь координат будет задаваться системой линейных уравнений:

с1(1)=c~1(1)e1(1)+c~2(1)e1(2)+…+c~n(1)e1(n)с2(1)=c~1(1)e2(1)+c~2(1)e2(2)+…+c~n(1)e2(n)⋮                                                           сn(1)=c~1(1)en(1)+c~2(1)en(2)+…+c~n(1)en(n)

В виде матрицы систему можно отобразить так:

(c1(1), c2(1),…, cn(1))=(c~1(1), c~2(1),…, c~n(1))·e1(1)e2(1)…en(1)e1(2)e2(2)…en(2)⋮⋮⋮⋮e1(n)e2(n)…en(n)

Сделаем по аналогии такую же запись для вектора c(2):

(c1(2), c2(2),…, cn(2))=(c~1(2), c~2(2),…, c~n(2))·e1(1)e2(1)…en(1)e1(2)e2(2)…en(2)⋮⋮⋮⋮e1(n)e2(n)…en(n)

И, далее действуя по тому же принципу, получаем:

(c1(n), c2(n),…, cn(n))=(c~1(n), c~2(n),…, c~n(n))·e1(1)e2(1)…en(1)e1(2)e2(2)…en(2)⋮⋮⋮⋮e1(n)e2(n)…en(n)

Матричные равенства объединим в одно выражение:

c1(1)c2(1)⋯cn(1)c1(2)c2(2)⋯cn(2)⋮⋮⋮⋮c1(n)c2(n)⋯cn(n)=c~1(1)c~2(1)⋯c~n(1)c~1(2)c~2(2)⋯c~n(2)⋮⋮⋮⋮c~1(n)c~2(n)⋯c~n(n)·e1(1)e2(1)⋯en(1)e1(2)e2(2)⋯en(2)⋮⋮⋮⋮e1(n)e2(n)⋯en(n)

Оно и будет определять связь векторов двух различных базисов.

Используя тот же принцип, возможно выразить все векторы базиса e(1), e(2),…, e(3) через базис c(1), c(2),…, c(n):

e1(1)e2(1)⋯en(1)e1(2)e2(2)⋯en(2)⋮⋮⋮⋮e1(n)e2(n)⋯en(n)=e~1(1)e~2(1)⋯e~n(1)e~1(2)e~2(2)⋯e~n(2)⋮⋮⋮⋮e~1(n)e~2(n)⋯e~n(n)·c1(1)c2(1)⋯cn(1)c1(2)c2(2)⋯cn(2)⋮⋮⋮⋮c1(n)c2(n)⋯cn(n)

Дадим следующие определения:

Определение 5

Матрица c~1(1)c~2(1)⋯c~n(1)c~1(2)c~2(2)⋯c~n(2)⋮⋮⋮⋮c~1(n)c~2(n)⋯c~n(n) является матрицей перехода от базиса e(1), e(2),…, e(3)

к базису c(1), c(2),…, c(n).

Определение 6

Матрица e~1(1)e~2(1)⋯e~n(1)e~1(2)e~2(2)⋯e~n(2)⋮⋮⋮⋮e~1(n)e~2(n)⋯e~n(n) является матрицей перехода от базиса c(1), c(2),…, c(n)

к базису e(1), e(2),…, e(3).

Из этих равенств очевидно, что

c~1(1)c~2(1)⋯c~n(1)c~1(2)c~2(2)⋯c~n(2)⋮⋮⋮⋮c~1(n)c~2(n)⋯c~n(n)·e~1(1)e~2(1)⋯e~n(1)e~1(2)e~2(2)⋯e~n(2)⋮⋮⋮⋮e~1(n)e~2(n)⋯e~n(n)=10⋯001⋯0⋮⋮⋮⋮00⋯1e~1(1)e~2(1)⋯e~n(1)e~1(2)e~2(2)⋯e~n(2)⋮⋮⋮⋮e~1(n)e~2(n)⋯e~n(n)·c~1(1)c~2(1)⋯c~n(1)c~1(2)c~2(2)⋯c~n(2)⋮⋮⋮⋮c~1(n)c~2(n)⋯c~n(n)=10⋯001⋯0⋮⋮⋮⋮00⋯1 

т.е. матрицы перехода взаимообратны.

Рассмотрим теорию на конкретном примере.

Пример 7

Исходные данные: необходимо найти матрицу перехода от базиса

c(1)=(1, 2, 1)c(2)=(2, 3, 3)c(3)=(3, 7, 1)

к базису

e(1)=(3, 1, 4)e(2)=(5, 2, 1)e(3)=(1, 1, -6)

Также нужно указать связь координат произвольного вектора x→ в заданных базисах.

Решение

1. Пусть T – матрица перехода, тогда верным будет равенство:

314521111=T·121233371

Умножим обе части равенства на

121233371-1

и получим:

T=31452111-6·121233371-1

2. Определим матрицу перехода:

T=31452111-6·121233371-1==31452111-6·-18537-2-15-1-1=-2794-712012-4198

3. Определим связь координат вектора x→:

допустим, что в базисе c(1), c(2),…, c(n) вектор x→ имеет координаты x1,x2,x3, тогда:

x=(x1,x2,x3)·121233371,

а в базисе e(1), e(2),…, e(3) имеет координаты x~1,x~2,x~3, тогда:

x=(x~1,x~2,x~3)·31452111-6

Т.к. равны левые части этих равенств, мы можем приравнять и правые:

(x1,x2,x3)·121233371=(x~1,x~2,x~3)·31452111-6

Умножим обе части справа на

121233371-1

и получим:

(x1,x2,x3)=(x~1,x~2,x~3)·31452111-6·121233371-1⇔⇔(x1,x2,x3)=(x~1,x~2,x~3)·T⇔⇔(x1,x2,x3)=(x~1,x~2,x~3)·-2794-712012-4198

С другой стороны

(x~1,x~2,x~3)=(x1,x2,x3)·-2794-712012-4198

Последние равенства показывают связь координат вектора x→ в обоих базисах.

Ответ: матрица перехода

-2794-712012-4198

Координаты вектора x→ в заданных базисах связаны соотношением:

(x1,x2,x3)=(x~1,x~2,x~3)·-2794-712012-4198

или

(x~1,x~2,x~3)=(x1,x2,x3)·-2794-712012-4198-1

При проведении научных и прикладных исследование часто создаются модели, в которых рассматриваются точки и/или векторы определенных пространств. Например, в моделях шифров на эллиптических кривых используются аффинные и проективные пространства. К проективным прибегают тогда, когда необходимо ускорить вычисления, так как в формулах манипулирования с точками эллиптической кривой выводимых в рамках проективного пространства отсутствует операция деления на координату, которую в случае аффинного пространства обойти не удается.

Операция деления как раз одна из самых «дорогих» операций. Дело в том, что в алгебраических полях, а соответственно и в группах операция деления вообще отсутствует и выход из положения (когда не делить нельзя) состоит в том, что операцию деления заменяют умножением, но умножают не на саму координату, а на обращенное ее значение. Из этого следует, что предварительно надо привлекать расширенный алгоритм Евклида НОД и кое что еще. Одним словом, не все так просто как изображают авторы большинства публикаций о ЕСС. Почти все, что по этой теме опубликовано и не только в Интернете мне знакомо. Мало того, что авторы не компетентны и занимаются профанацией, оценщики этих публикаций плюсуют авторов в комментариях, т. е. не видят ни пробелов, ни явных ошибок. Про нормальную же статью пишут, что она уже 100500-я и от нее нулевой эффект. Так все пока на Хабре устроено, анализ публикаций делается огромный, но не качества содержания. Здесь возразить нечего — реклама двигатель бизнеса.

Линейное векторное пространство

Изучение и описание явлений окружающего мира с необходимостью приводит нас к введению и использованию ряда понятий таких как точки, числа, пространства, прямые линии, плоскости, системы координат, векторы, множества и др.

Пусть r<3> = <r1, r2, r3> вектор трехмерного пространства, задает положение одной частицы (точки) относительно начала координат. Если рассматривать N элементов, то описание их положения требует задания 3∙N координат, которые можно рассматривать как координаты некоторого вектора в 3N-мерном пространстве. Если рассматривать непрерывные функции и их совокупности, то приходим к пространствам, размерность которых равна бесконечности. На практике часто ограничиваются использованием лишь подпространства такого бесконечномерного пространства функции координат, обладающего конечным числом измерений.

Пример 1. Ряд Фурье — пример использования пространства функций. Рассмотрим разложение произвольной функции в ряд Фурье

Его можно трактовать как разложение «вектора» f(x) по бесконечному набору «ортогональных» базисных векторов sinпх

Это пример абстрагирования и распространения понятия вектора на бесконечное число измерений. Действительно, известно, что при -π≤x≤π

Существо дальнейшего рассмотрения не пострадает, если мы отвлечемся от размерности абстрактного векторного пространства – будь — то 3, 3N или бесконечность, хотя для практических приложений больший интерес представляет конечномерные поля и векторные пространства.

Набор векторов r1, r2,… будем называть линейным векторным пространством L, если сумма любых двух его элементов тоже находится в этом наборе и если результат умножения элемента на число С также входит в этот набор. Оговоримся сразу, что значения числа С могут быть выбраны из вполне определенного числового множества Fр – поля вычетов по модулю простого числа р, которое считается присоединенным к L.

Пример 2. Набор из 8 векторов, составленных из n =5 -разрядных двоичных чисел
r0 = 00000, r1 = 10101, r2 = 01111, r3 = 11010, r4 = 00101, r5 = 10110, r6 = 01001, r7 = 11100 образует векторное пространство L, если числа С є {0,1}. Этот небольшой пример позволяет убедиться в проявлении свойств векторного пространства, включенных в его определение.

Суммирование этих векторов выполняется поразрядно по модулю два, т. е. без переноса единиц в старший разряд. Отметим, что если все С действительные (в общем случае С принадлежат полю комплексных чисел), то векторное пространство называют действительным.

Формально аксиомы векторного пространства и записываются так:
r1 + r2 = r2 + r1 = r3; r1, r2, r3 є L – коммутативность сложения и замкнутость;
(r1 + r2) + r3 = r1 + (r2 + r3) = r1 + r2 + r3 – ассоциативность сложения;
ri + r0 = r0 + ri = ri; ∀i, ri, r0 є L–существование нейтрального элемента;
ri +(- ri) = r0, для ∀i существует противоположный вектор (-ri) є L;
1∙ ri = ri ∙1 = ri существование единицы для умножения;
α (β∙ri) = (α∙β)∙ri; α, β, 1, 0 – элементы числового поля F, ri є L; умножение на скаляры ассоциативно; результат умножения принадлежит L;
(α + β) ri = α∙ri + β∙ri; для ∀i, ri є L, α, β – скаляры;
а (ri + rj) = ari + arj для всех а, ri, rj є L;
a∙0 = 0, 0∙ri = 0; (-1) ∙ ri = – ri.

Размерность и базис векторного пространства

При изучении векторных пространств представляет интерес выяснение таких вопросов, как число векторов, образующих все пространство; какова размерность пространства; какой наименьший набор векторов путем применения к нему операции суммирования и умножения на число позволяет сформировать все векторы пространства? Эти вопросы основополагающие и их нельзя обойти стороной, так как без ответов на них утрачивается ясность восприятия всего остального, что составляет теорию векторных пространств.

Оказалось, что размерность пространства самым тесным образом связана с линейной зависимостью векторов, и с числом линейно независимых векторов, которые можно выбирать в изучаемом пространстве многими способами.

Линейная независимость векторов

Набор векторов r1, r2, r3 … rр из L называют линейно независимым, если для них соотношение

выполняется только при условии одновременного равенства

$с_1=с_2=…=с_р=0$.
Все

$с_k$, k = 1(1)p, принадлежат числовому полю вычетов по модулю два
F = {0, 1}.
Если в некотором векторном пространстве L можно подобрать набор из р векторов, для которых соотношение

$c_1 r_1+c_2 r_2+...+c_p r_p=0 $ выполняется, при условии, что не все

$с_k = 0$ одновременно, т.е. в поле вычетов оказалось возможным выбрать набор

$с_k$, k =1(1)р, среди которых есть ненулевые, то такие векторы

$r_i$ называются линейно зависимыми.

Пример 3. На плоскости два вектора

$е_1$ = <0, 1>T и

$е_2$ = <1, 0>T являются линейно независимыми, так как в соотношении (T-транспонирование)

невозможно подобрать никакой пары чисел

$с_1, с_2$ коэффициентов не равных нулю одновременно, чтобы соотношение было выполнено.
Три вектора

$е_1$ = <0, 1>T,

$е_2$ = <1, 0>T,

$е_3$ = <1, 1>T образуют систему линейно зависимых векторов, так как в соотношении

равенство может быть обеспечено выбором коэффициентов

$с_1 = с_2 = 1, с_3 = –1$, не равных нулю одновременно. Более того, вектор

$ e_3 = е_1 + е_2 $ является функцией

$ е_1$ и

$ е_2 $ (их суммой), что указывает на зависимость

$ e_3 $ от

$е_1$ и

$е_2 $. Доказательство общего случая состоит в следующем.

Пусть хотя бы одно из значений

$с_k$, k = 1(1)р, например,

$с_р ≠ 0$, а соотношение выполнено. Это означает, что векторы

$r_k$, k = 1(1)р, линейно зависимы

Выделим явным образом из суммы вектор rр

Говорят, что вектор rр является л и н е й н о й комбинацией векторов

$r_1, r_2… r_р-1$ или rр через остальные векторы выражается линейным образом, т.е. rр линейно зависит от остальных. Он является их функцией.

На плоскости двух измерений любые три вектора линейно зависимы, но любые два неколлинеарных вектора являются независимыми. В трехмерном пространстве любые три некомпланарных вектора линейно независимы, но любые четыре вектора всегда линейно зависимы.

Зависимость/независимость совокупности {

${e_1, e_2, e_3, ..., e_n}$} векторов часто определяют, вычисляя определитель матрицы Грама (ее строки скалярные произведения наших векторов). Если определитель равен нулю, среди векторов имеются зависимые, если определитель отличен от нуля — векторы в матрице независимы.

Определителем Грама (грамианом) системы векторов

$ {displaystyle mathbf {e} _{1},;mathbf {e} _{2},;ldots ,mathbf {e} _{n}}mathbf{e}_1,;mathbf{e}_2,;ldots,mathbf{e}_n$

в евклидовом пространстве называется определитель матрицы Грама этой системы:

${displaystyle {begin{vmatrix}langle e_{1},;e_{1}rangle &langle e_{1},;e_{2}rangle &ldots &langle e_{1},;e_{n}rangle \langle e_{2},;e_{1}rangle &langle e_{2},;e_{2}rangle &ldots &langle e_{2},;e_{n}rangle \ldots &ldots &ldots &ldots \langle e_{n},;e_{1}rangle &langle e_{n},;e_{2}rangle &ldots &langle e_{n},;e_{n}rangle \end{vmatrix}},}begin{vmatrix} langle e_1,;e_1rangle & langle e_1,;e_2rangle & ldots & langle e_1,;e_nrangle \ langle e_2,;e_1rangle & langle e_2,;e_2rangle & ldots & langle e_2,;e_nrangle \ ldots & ldots & ldots & ldots \ langle e_n,;e_1rangle & langle e_n,;e_2rangle & ldots & langle e_n,;e_nrangle \ end{vmatrix},$

где

${displaystyle langle e_{i},;e_{j}rangle }langle e_i,;e_jrangle{displaystyle langle e_{i},;e_{j}rangle }langle e_i,;e_jrangle$ — скалярное произведение векторов

${displaystyle mathbf {e} _{i}}mathbf{e}_i$ и

${displaystyle mathbf {e} _{j}}mathbf{e}_j$.

Размерность и базис векторного пространства

Размерность s = d (L) пространства L определяется как наибольшее число векторов в L, образующих линейно независимый набор. Размерность – это не число векторов в L, которое может быть бесконечным и не число компонентов вектора.

Пространства, имеющие конечную размерность s ≠ ∞, называются конечномерными, если
s = ∞, – бесконечномерными.

Ответом на вопрос о минимальном числе и составе векторов, которые обеспечивают порождение всех векторов линейного векторного пространства является следующее утверждение.

Любой набор s линейно независимых векторов в пространстве L образует его б а з и с. Это следует из того, что любой вектор

$r_k$ линейного s-мерного векторного пространства L может быть представлен единственным способом в виде линейной комбинации векторов базиса.

Зафиксируем и обозначим символом

$е_i $, i = 1(1)s, один из наборов, образующих базис пространства L. Тогда

Числа rki, i = 1(1)s называются координатами вектора

$r_k $ в базисе

$е_i $, i = 1(1)s, причем rki = (

$е_i $,

$r_k $).
Покажем единственность представления

$r_k $. Очевидно, что набор

$e_1,e_2,...,e_s$,

$ r_k $ является зависимым, так как

$е_i$, i = 1(1)s – базис. Другими словами, существуют такие

$с_1, с_2... с_s, c_k$ не равные одновременно нулю, что

$ c_1·e_1 + c_2·e_2 + ...+ c_s·e_s + c_k·r_k = 0$.
При этом пусть

$c_k ≠ 0$, ибо если

$ c_k = 0 $, то хоть одно из

$с_1, с_2 , ... , с_s$, было бы отлично от нуля и тогда векторы

$ e_i $, i = 1(1)s, были бы линейно зависимы, что невозможно, так как это базис. Следовательно,

Полагая

, будем иметь
Используя прием доказательства «от противного», допустим, что записанное представление

$r_k $ не единственное в этом базисе и существует другое

Тогда запишем отличие представлений, что, естественно, выражается как

Очевидно, что правая и левая части равны, но левая представляет разность вектора с самим собой, т. е. равна нулю. Следовательно, и правая часть равна нулю. Векторы

$ е_i $, i = 1(1)s линейно независимы, поэтому все коэффициенты при них могут быть только нулевыми. Отсюда получаем, что

а это возможно только при

Выбор базиса. Ортонормированность

Векторы называют нормированными, если длина каждого из них равна единице. Этого можно достичь, применяя к произвольным векторам процедуру нормировки.

Векторы называют ортогональными, если они перпендикулярны друг другу. Такие векторы могут быть получены применением к каждому из них процедуры ортогонализации. Если для совокупности векторов выполняются оба свойства, то векторы называются ортонормированными.

Необходимость рассмотрения ортонормированных базисов вызвана потребностями использования быстрых преобразований как одно –, так и многомерных функций. Задачи такой обработки возникают при исследовании кодов, кодирующих информационные сообщения в сетях связи различного назначения, при исследовании изображений, получаемых
посредством автоматических и автоматизированных устройств, в ряде других областей, использующих цифровые представления информации.

Определение. Совокупность n линейно независимых векторов n-мерного векторного
пространства V называется его базисом.

Теорема. Каждый вектор х линейного n-мерного векторного пространства V можно представить, притом единственным образом, в виде линейной комбинации векторов базиса. Векторное пространство V над полем F обладает следующими свойствами:
0·х = 0 (0 в левой части равенства – нейтральный элемент аддитивной группы поля F; 0 в правой части равенства – элемент пространства V, являющийся нейтральным единичным элементом аддитивной группы V, называемый нулевым вектором);
(– 1)·х = –х; –1є F; x є V; –x є V;
Если α·х = 0єV, то при х ≠ 0 всегда α = 0.
Пусть Vn(F) – множество всех последовательностей (х1, х2, …, хn) длины n с компонентами из поля F, т.е. Vn(F) ={x, таких, что х = (х1, х2, …, хn), хi є F;
i =1(1)n }.

Сложение и умножение на скаляр определяются следующим образом:
x + y =(x1 + y1, x2 + y2, …, xn + yn);
α·х = (α·х1, α·х2,…, α·хn), где у = (у1, у2,…, уn),
тогда Vn(F) является векторным пространством над полем F.

Пример 4. В векторном пространстве rо = 00000, r1 = 10101, r2 = 11010, r3 = 10101 над полем F2 = {0,1} определить его размерность и базис.
Решение. Сформируем таблицу сложения векторов линейного векторного пространства

В этом векторном пространстве V= {rо,r1,r2,r3} каждый вектор в качестве противоположного имеет самого себя. Любые два вектора, исключая rо, являются линейно независимыми, в чем легко убедиться
c1·r1 + c2·r2 = 0; c1·r1 + c3·r3 = 0; c2·r2 + c3·r3 = 0;

Каждое из трех соотношений справедливо только при одновременных нулевых значениях пар коэффициентов сi, сj є {0,1}.

При одновременном рассмотрении трех ненулевых векторов один из них всегда является суммой двух других или равен самому себе, а r1+r2+r3=rо.

Таким образом, размерность рассматриваемого линейного векторного пространства равна двум s = 2, d(L) = s = 2, хотя каждый из векторов имеет пять компонентов. Базисом пространства является набор (r1, r2). Можно в качестве базиса использовать пару (r1, r3).

Важным в теоретическом и практическом отношении является вопрос описания векторного пространства. Оказывается, любое множество базисных векторов можно рассматривать как строки некоторой матрицы G, называемой порождающей матрицей векторного пространства. Любой вектор этого пространства может быть представлен как линейная комбинация строк матрицы G ( как, например, здесь).

Если размерность векторного пространства равна k и равна числу строк матрицы G, рангу матрицы G, то очевидно, существует k коэффициентов с q различными значениями для порождения всех возможных линейных комбинаций строк матрицы. При этом векторное пространство L содержит qk векторов.

Множество всех векторов из ℤpn с операциями сложения векторов и умножения вектора на скаляр из ℤp есть линейное векторное пространство.

Определение. Подмножество W векторного пространства V, удовлетворяющее условиям:
Если w1, w2 є W, то w1+ w2 є W,
Для любых α є F и w є W элемент αw є W,
само является векторным пространством над полем F и называется подпространством векторного пространства V.

Пусть V есть векторное пространство над полем F и множество W ⊆ V. Множество W есть подпространство пространства V, если W по отношению к линейным операциям, определенным в V, есть линейное векторное пространство.

Таблица. Характеристики векторных пространств

Компактность матричного представления векторного пространства очевидна. Например, задание L векторов двоичных 50-разрядных чисел, среди которых 30 векторов образуют базис векторного пространства, требует формирования матрицы G[30,50], а описываемое количество векторов превышает 109, что в поэлементной записи представляется неразумным.

Все базисы любого пространства L разбиваются подгруппой Р невырожденных матриц с det G > 0 на два класса. Один из них (произвольно) называют классом с положительно ориентированными базисами (правыми), другой класс содержит левые базисы.

В этом случае говорят, что в пространстве задана ориентация. После этого любой базис представляет собой упорядоченный набор векторов.

Если нумерацию двух векторов изменить в правом базисе, то базис станет левым. Это связано с тем, что в матрице G поменяются местами две строки, следовательно, определитель detG изменит знак.

Норма и скалярное произведение векторов

После того как решены вопросы о нахождении базиса линейного векторного пространства, о порождении всех элементов этого пространства и о представлении любого элемента и самого векторного пространства через базисные векторы, можно поставить задачу об измерении в этом пространстве расстояний между элементами, углов между векторами, значений компонентов векторов, длины самих векторов.

Действительное или комплексное векторное пространство L называется нормированным векторным пространством, если каждый вектор r в нем может быть сопоставлен действительному числу || r || – модулю вектора, норме. Единичный вектор – это вектор, норма которого равна единице. Нулевой вектор имеет компонентами нули.

Определение. Векторное пространство называется унитарным, если в нем определена бинарная операция, ставящая каждой паре ri, rj векторов из L в соответствие скаляр. В круглых скобках (ri, rj) записывается (обозначается) скалярное или внутреннее произведение ri и rj, причем
1. (ri, rj) = ri ∙ rj;
2. (ri, rj) = (rj ∙ ri)*, где * указывает на комплексное сопряжение или эрмитову симметрию;
3. (сri, rj) = с(ri ∙ rj) – ассоциативный закон;
4. (ri + rj, rk) = (ri ∙ rk)+ (rj ∙ rk)– дистрибутивный закон;
5. (ri, rk) ≥ 0 и из (ri, rj ) = 0 следует ri = 0.

Определение. Положительное значение квадратного корня называют нормой (или длиной, модулем) вектора ri. Если = 1, то вектор ri называют нормированным.

Два вектора ri, rj унитарного векторного пространства L взаимно ортогональны, если их скалярное произведение равно нулю, т.е. (ri, rj) = 0.

При s = 3 в линейном векторном пространстве в качестве базиса удобно выбирать три взаимно перпендикулярных вектора. Такой выбор существенно упрощает ряд зависимостей и вычислений. Этот же принцип ортогональности используется при выборе базиса в пространствах и других размерностей s > 3. Использование введенной операции скалярного произведения векторов обеспечивает возможность такого выбора.

Еще большие преимущества достигаются при выборе в качестве базиса векторного пространства ортогональных нормированных векторов – ортонормированного базиса. Если не оговорено специально, то далее всегда будем считать, что базис еi, i = 1(1)s выбран именно таким образом, т.е.

, где ij — символ Кронекера (1823 — 1891).

В унитарных векторных пространствах такой выбор всегда реализуем. Покажем реализуемость такого выбора.

Определение. Пусть S = {v1, v2,…, vn} есть конечное подмножество векторного пространства V над полем F.
Линейная комбинация векторов из S есть выражение вида а1∙v1 + а2∙v2 +…+ аn∙vn, где каждое аi ∊ F.

Оболочка для множества S (обозначение {S}) есть множество всех линейных комбинаций векторов из S. Оболочка для S есть подпространство пространства V.

Если U есть пространство в V, то U натянуто на S (S стягивает U), если {S}=U.
Множество векторов S линейно зависимо над F, если в F существуют скаляры а1, а2,…, аn, не все нули, для которых а1∙v1+ а2∙v2 +…+ аn∙vn = 0. Если таких скаляров не существует, то множество векторов S линейно независимо над F.

Если векторное пространство V натянуто на линейно независимую систему векторов S (или система S стягивает пространство V), то система S называется базисом для V.

Приведение произвольного базиса к ортонормированному виду

Пусть в пространстве V имеется не ортонормированный базис ē i, i = 1(1)s. Обозначим норму каждого вектора базиса символом

Известно следующее утверждение [11]. Если ē i, i = 1(1)s – произвольная конечная или счетная система линейно независимых векторов в унитарном векторном пространстве, то существует ортонормированная система ē i, i = 1(1)s, порождающая то же самое линейное пространство (многообразие).

В основу процедуры приведения базиса к ортонормированному виду положен процесс ортогонализации Грама — Шмидта, который в свою очередь, реализуется рекуррентными формулами

В развернутом виде алгоритм ортогонализации и нормирования базиса содержит следующие условия:

Делим вектор ē 1, на его норму; получим нормированный вектор ē i 1/(||ē 1 ||);
Формируем V2 = ē 2 — (ē 1, ē 2)e 1 и нормируем его, получим е 2. Ясно, что тогда
(е1, е2) ~ (е1, е2) – (е1, ē 2)( е1, е1) = 0;
Построив V3 = ē 3– (e1, ē 3)e1 – (e2, ē 3) e2 и нормируя его, получим е3.

Для него имеем сразу же (е1, е3) = (е2, е3) = 0.
Продолжая такой процесс, получим ортонормированный набор ē i, i = 1(1)s. Этот набор содержит линейно независимые векторы, поскольку все они взаимно ортогональны.
Убедимся в этом. Пусть выполняется соотношение

Если набор ē i, i = 1(1)s зависимый, то хотя бы один сj коэффициент не равен нулю сj ≠ 0.

Умножив обе части соотношения на еj, получаем
(ej, c1∙e1 ) + (ej, c2∙e2 )+ …+ ( ej, cj∙ej ) +…+ ( ej, cs∙rs ) = 0.
Каждое слагаемое в сумме равно нулю как скалярное произведение ортогональных векторов, кроме (ej ,cj∙ej), которое равно нулю по условию. Но в этом слагаемом
(ej, ej) = 1 ≠ 0, следовательно, нулем может быть только cj.
Таким образом, допущение о том, что cj ≠ 0 неверно и набор является линейно независимым.

Пример 5. Задан базис 3-х мерного векторного пространства:
{<-1, 2 ,3, 0>,<0, 1, 2, 1>,<2,-1,-1,1>}.
Скалярное произведение определено соотношением:
(<x1,x2,x3,x4>,<y1,y2,y3,y4>) = x1∙y1+x2∙y2+x3∙y3+x4∙y4.
Процедурой ортогонализации Грама — Шмидта получаем систему векторов:
а1 = <-1, 2, 3, 0>; a2 = <0, 1, 2, 1>-4<-1, 2, 3,0>/7=<4,-1, 2, 7>/7;
a3 =<2, -1, -1, 1>+½<-1, 2, 3, 0> — <4, -1, 2, 7>/5 =<7, 2, 1, -4>/10.
(a1,a2)= (1+4+9+0) = 14;
a1 E =a1/√14;
a2-(a1E,a2)∙a1E=a2-(8/√14)(a1/√14)=a2 — 4∙a1/7;
Третий вектор читателю предлагается обработать самостоятельно.

Нормированные векторы получают вид:
a1 E =a1/√14;
a2 E =<4, -1, 2, 7>/√70;
a3 E =<7, 2, 1,-4>/√70;

Ниже в примере 6 дается подробный развернутый процесс вычислений получения ортонормированного базиса из простого (взятого наугад).

Пример 6. Привести заданный базис линейного векторного пространства к ортонормированному виду.
Дано: векторы базиса

Подпространства векторных пространств

Структура векторного пространства

Представление объектов (тел) в многомерных пространствах весьма непростая задача. Так, четырехмерный куб в качестве своих граней имеет обычные трехмерные кубы, и в трехмерном пространстве может быть построена развертка четырехмерного куба. В некоторой степени «образность» и наглядность объекта или его частей способствует более успешному его изучению.

Сказанное позволяет предположить, что векторные пространства можно некоторым образом расчленять, выделять в них части, называемые подпространствами. Очевидно, что рассмотрение многомерных и тем более бесконечномерных пространств и объектов в них лишает нас наглядности представлений, что весьма затрудняет исследование объектов в таких
пространствах. Даже, казалось бы, такие простые вопросы, как количественные характеристики элементов многогранников (число вершин, ребер, граней, и т. п.) в этих пространствах решены далеко не полностью.

Конструктивный путь изучения подобных объектов состоит в выделении их элементов (например, ребер, граней) и описании их в пространствах меньшей размерности. Так четырехмерный куб в качестве своих граней имеет обычные трехмерные кубы и в трехмерном пространстве может быть построена развертка четырехмерного куба. В некоторой степени
«образность» и наглядность объекта или его частей способствует более успешному их изучению.

Если L – расширение поля К, то L можно рассматривать как векторное (или линейное) пространство над полем К. Элементы поля L (т. е. векторы) образуют по сложению абелеву группу. Кроме того, каждый «вектор» а є L может быть умножен на «скаляр» r є K, и при этом произведение ra снова принадлежит L (здесь ra – просто произведение в смысле операции поля L элементов r и а этого поля). Выполняются также законы
r∙(a+b) = r∙a+r∙b, (r+s)∙a = r∙a + r∙s, (r∙s)∙a = r∙(s∙a) и 1∙а = а, где r,s є K, a,b є L.

Сказанное позволяет предположить, что векторные пространства можно некоторым образом расчленять, выделять в них части, называемые подпространствами. Очевидно, что основным результатом при таком подходе является сокращение размерности выделяемых подпространств. Пусть в векторном линейном пространстве L выделены подпространства L1 и L2. В качестве базиса L1 выбирается меньший набор еi, i = 1(1)s1, s1 < s, чем в исходном L.

Оставшиеся базисные векторы порождают другое подпространство L2, называемое «ортогональным дополнением» подпространства L1. Будем использовать запись L = L1 + L2. Она означает не то, что все векторы пространства L принадлежат либо L1, либо L2,, а то, что любой вектор из L можно представить в виде суммы вектора из L1 и ортогонального ему вектора из L2.
Разбивается не множество векторов векторного пространства L, а размерность d(L) и набор базисных векторов. Таким образом, подпространством L1 векторного пространства L называется множество L1, его элементов (меньшей размерности), само являющееся векторным пространством относительно введенных в L операций сложения и умножения на число.

Каждое линейное векторное подпространство Li – содержит нулевой вектор и вместе с любыми своими векторами содержит и все их линейные комбинации. Размерность любого линейного подпространства не превосходит размерности самого исходного пространства.

Пример 7. В обычном трехмерном пространстве подпространствами являются все прямые (размерность s =1) линии, плоскости (размерность s = 2), проходящие через начало координат. В пространстве Рn многочленов степени не выше n подпространствами будут, например, все Рk при k < n, так как складывая и умножая на числа многочлены степени, не выше k, снова будут получаться такие же многочлены.

Однако, каждое из пространств Рп содержится в качестве подпространств в пространстве Р всех многочленов с вещественными коэффициентами, а это последнее является подпространством пространства С непрерывных функций.

Матрицы одинакового типа над полем действительных чисел также образуют линейное векторное пространство, так как для них выполняются все аксиомы векторных пространств. Векторное пространство L2 наборов длины n, каждый из которых ортогонален подпространству L1 наборов длины п, образует подпространство L2, называемое нулевым пространством для L1. Другими словами, каждый вектор из L2 ортогонален каждому вектору из L1 и наоборот.

Оба подпространства L1 и L2 являются подпространствами векторного пространства L наборов длины п. В теории кодирования [4] каждое из подпространств L1 и L2 порождает линейный код, двойственный по отношению к коду, порожденному другими подпространства-ми. Если L1 есть (п, k)-код, то L2 — это (п, п – k)-код. Если код является векторным пространством строк некоторой матрицы, то двойственный к нему код — нулевое пространство этой матрицы и наоборот.

Важным вопросом при изучении векторных пространств Vn является установление их структуры (строения). Другими словами, интерес представляют элементы, их совокупности (подпространства размерности 1<k<п ), а также их отношения (упорядоченность, вложенность и т.п.). Будем считать заданным векторное пространство Vn над конечным полем GF(q), образованным q = р r элементами, где р — простое число, r — целое.
Известны следующие результаты.

Количества подпространств векторного пространства

Приведем следующее обоснование. Каждый вектор v1 ≠ 0 из системы k линейно независимых ( v1,v2,…,vk ) векторов может быть выбран qn – 1 способами. Следующий вектор v2 ≠ 0 не может быть выражен линейно через v1, т.е. может быть выбран qn – q способами и т.д.

Последний вектор vk ≠ 0 также линейно не выражается через предыдущие выбранные векторы v1,v2,…,vk и, следовательно, может быть выбран qn – qk – 1 способами. Общее число способов для выбора совокупности векторов v1,v2,…,vk, таким образом, определится как произведение числа выборов отдельных векторов, что и дает формулу (1). Для случая, когда k = п, имеем wп = wn, n и из формулы (I) получаем формулу (2).

Важные обобщающие результаты о размерностях подпространств.
Совокупность всех наборов длины n, ортогональных подпространству V1 наборов длины n, образует подпространство V2 наборов длины n. Это подпространство V2 называется нулевым пространством для V1.
Если вектор ортогонален каждому из векторов, порождающих подпространство V1, то этот вектор принадлежит нулевому пространству для V1.
Примером (V1) может служить множество 7-разрядных векторов порождающей матрицы (7,4)-кода Хемминга, с нулевым подпространством (V2) 7-разрядных векторов, образующих проверочную матрицу этого кода.

Если размерность подпространства (V1) наборов длины n равна k, то размерность нулевого подпространства (V2) равна n — k.

Если V2 — подпространство наборов длины n и V1 — нулевое пространство для V2, то (V2) — нулевое пространство для V1.

Пусть U∩V обозначает совокупность векторов, принадлежащих одновременно U и V, тогда U∩V является подпространством.

Пусть U⊕V обозначает подпространство, состоящее из совокупности всех линейных комбинаций вида au +bv, где u є U, v є V, a b — числа.

Сумма размерностей подпространств U∩V и U⊕V равна сумме размерностей подпространств U и V.

Пусть U2 — нулевое подпространство для U1, а V2 -нулевое пространство для V1. Тогда U2∩V2 является нулевым пространством для U1⊕V1.

Заключение

В работе рассмотрены основные понятия векторных пространств, которые часто используются при построении моделей анализа систем шифрования, кодирования и стеганографических, процессов, протекающих в них. Так в новом американском стандарте шифрования использованы пространства аффинные, а в цифровых подписях на эллиптических кривых и аффинные и
проективные (для ускорения обработки точек кривой).

Об этих пространствах в работе речь не идет (нельзя валить все в одну кучу, да и объем публикации я ограничиваю), но упоминания об этом сделаны не зря. Авторы, пишущие о средствах защиты, об алгоритмах шифров наивно полагают, что понимают детали описываемых явлений, но понимание евклидовых пространств и их свойств без всяких оговорок переносится в другие пространства, с другими свойствами и законами. Читающая аудитория вводится в заблуждение относительно простоты и доступности материала.

Создается ложная картина действительности в области информационной безопасности и специальной техники (технологий и математики).

В общем почин мною сделан, насколько удачно судить читателям.

Литература

1. Авдошин С.М., Набебин А.А. Дискретная математика. Модулярная алгебра, криптография, кодирование. — М.: ДМК Пресс, 2017. -352 с.
2. Акимов О.Е. Дискретная математика.Логика, группы, графы- М.: Лаб.Баз. Зн., 2001. -352 с.
3. Андерсон Д.А. Дискретная математика и комбинаторика.- М.: Вильямс, 2003. -960 с.
4. Берлекэмп Э. Алгебраическая теория кодирования. -М.: Мир,1971.- 478 с.
5. Ваулин А.Е. Дискретная математика в задачах компьютерной безопасности. Ч 1- СПб.: ВКА им. А.Ф. Можайского, 2015. -219 с.
6. Ваулин А.Е. Дискретная математика в задачах компьютерной безопасности. Ч 2- СПб.: ВКА им. А.Ф. Можайского, 2017. -151 с.
7. Горенстейн Д. Конечные простые группы. Введение в их классификацию.-М.: Мир,1985.- 352 с.
7. Грэхем Р., Кнут Д., Пташник О. Конкретная математика.Основание информатики.-М.: Мир,1998.-703 с.
9. Елизаров В.П. Конечные кольца.- М.: Гелиос АРВ,2006. — 304 с.
Иванов Б.Н. Дискретная математика: алгоритмы и программы-М.: Лаб.Баз. Знаний., 2001. -280 с.
10. Ерусалимский Я.М. Дискретная математика: теория, задачи, приложения-М.: Вузовская книга, 2000.-280 с.
11. Корн Г., Корн Т. Справочник по математике для научных работников и инженеров.-М.: Наука, 1973.-832 с.
12. Лидл Р., Нидеррайтер Г. Конечные поля: В 2-х т. Т.1 -М.: Мир,1988. — 430 с.
13. Лидл Р., Нидеррайтер Г. Конечные поля: В 2-х т. Т.2 -М.: Мир,1988. — 392 с.
14. Ляпин Е.С., АйзенштатА.Я., Лесохин М.М., Упражнения по теории групп.- М.: Наука,1967.-264 с.
15. Муттер В.М. Основы помехоустойчивой телепередачи информации. -Л. Энергоатомиздат,1990.- 288 с.
16. Набебин А.А.Дискретная математика.- М.: Лаб.Баз. Знаний., 2001. -280 с.
17. Новиков Ф.А. Дискретная математика для программистов.- СПб.: Питер, 2000. -304 с.
18. Розенфельд Б.А. Многомерные пространства.-М.: Наука,1966.-648 с.
18. Холл М. Теория групп.-М.: Изд. ИЛ, 1962.- 468 с.
19. Шиханович Ю.А. Группы, кольца, решётки. — СПб.: Кирцидели,2006. — 368 с.
20. Шнеперман Л.Б. Курс алгебры и теории чисел в задачах и упражнениях: В 2-х ч Ч.2.-Мн.: Выш. шк., 1987. -256 с.
21. Шнеперман Л.Б. Сборник задач по алгебре и теории чисел.- Минск: Дизайн ПРО, 2000. -240 с.

Векторное пространство: размерность и базис, разложение вектора по базису

В статье о n -мерных векторах мы пришли к понятию линейного пространства, порождаемого множеством n -мерных векторов. Теперь нам предстоит рассмотреть не менее важные понятия, такие как размерность и базис векторного пространства. Они напрямую связаны с понятием линейно независимой системы векторов, так что дополнительно рекомендуется напомнить себе основы и этой темы.

Введем некоторые определения.

Размерность векторного пространства – число, соответствующее максимальному количеству линейно независимых векторов в этом пространстве.

Базис векторного пространства – совокупность линейно независимых векторов, упорядоченная и в своей численности равная размерности пространства.

Рассмотрим некое пространство n -векторов. Размерность его соответственно равна n . Возьмем систему из n -единичных векторов:

e ( 1 ) = ( 1 , 0 , . . . , 0 ) e ( 2 ) = ( 0 , 1 , . . . , 0 ) e ( n ) = ( 0 , 0 , . . . , 1 )

Используем эти векторы в качестве составляющих матрицы A : она будет являться единичной с размерностью n на n . Ранг этой матрицы равен n . Следовательно, векторная система e ( 1 ) , e ( 2 ) , . . . , e ( n ) является линейно независимой. При этом к системе невозможно добавить ни одного вектора, не нарушив ее линейной независимости.

Так как число векторов в системе равно n , то размерность пространства n -мерных векторов равна n , а единичные векторы e ( 1 ) , e ( 2 ) , . . . , e ( n ) являются базисом указанного пространства.

Из полученного определения сделаем вывод: любая система n -мерных векторов, в которой число векторов меньше n , не является базисом пространства.

Если мы поменяем местами первый и второй вектор, получим систему векторов e ( 2 ) , e ( 1 ) , . . . , e ( n ) . Она также будет являться базисом n -мерного векторного пространства. Составим матрицу, взяв за ее строки векторы полученной системы. Матрица может быть получена из единичной матрицы перестановкой местами первых двух строк, ранг ее будет равен n . Система e ( 2 ) , e ( 1 ) , . . . , e ( n ) линейно независима и является базисом n -мерного векторного пространства.

Переставив местами в исходной системе другие векторы, получим еще один базис.

Мы можем взять линейно независимую систему неединичных векторов, и она также будет представлять собой базис n -мерного векторного пространства.

Векторное пространство с размерностью n имеет столько базисов, сколько существует линейно независимых систем из n -мерных векторов числом n.

Плоскость является двумерным пространством – ее базисом будут два любых неколлинеарных вектора. Базисом трехмерного пространства послужат три любых некомпланарных вектора.

Рассмотрим применение данной теории на конкретных примерах.

Исходные данные: векторы

a = ( 3 , – 2 , 1 ) b = ( 2 , 1 , 2 ) c = ( 3 , – 1 , – 2 )

Необходимо определить, являются ли указанные векторы базисом трехмерного векторного пространства.

Решение

Для решения поставленной задачи исследуем заданную систему векторов на линейную зависимость. Составим матрицу, где строки – координаты векторов. Определим ранг матрицы.

A = 3 2 3 – 2 1 – 1 1 2 – 2 A = 3 – 2 1 2 1 2 3 – 1 – 2 = 3 · 1 · ( – 2 ) + ( – 2 ) · 2 · 3 + 1 · 2 · ( – 1 ) – 1 · 1 · 3 – ( – 2 ) · 2 · ( – 2 ) – 3 · 2 · ( – 1 ) = = – 25 ≠ 0 ⇒ R a n k ( A ) = 3

Следовательно, заданные условием задачи векторы линейно независимы, и их численность равна размерности векторного пространства – они являются базисом векторного пространства.

Ответ: указанные векторы являются базисом векторного пространства.

Исходные данные: векторы

a = ( 3 , – 2 , 1 ) b = ( 2 , 1 , 2 ) c = ( 3 , – 1 , – 2 ) d = ( 0 , 1 , 2 )

Необходимо определить, может ли указанная система векторов являться базисом трехмерного пространства.

Решение

Указанная в условии задачи система векторов является линейно зависимой, т.к. максимальное число линейно независимых векторов равно 3. Таким образом, указанная система векторов не может служить базисом трехмерного векторного пространства. Но стоит отметить, что подсистема исходной системы a = ( 3 , – 2 , 1 ) , b = ( 2 , 1 , 2 ) , c = ( 3 , – 1 , – 2 ) является базисом.

Ответ: указанная система векторов не является базисом.

Исходные данные: векторы

a = ( 1 , 2 , 3 , 3 ) b = ( 2 , 5 , 6 , 8 ) c = ( 1 , 3 , 2 , 4 ) d = ( 2 , 5 , 4 , 7 )

Могут ли они являться базисом четырехмерного пространства?

Решение

Cоставим матрицу, используя в качестве строк координаты заданных векторов

A = 1 2 3 3 2 5 6 8 1 3 2 4 2 5 4 7

По методу Гаусса определим ранг матрицы:

A = 1 2 3 3 2 5 6 8 1 3 2 4 2 5 4 7

1 2 3 3 0 1 0 2 0 1 – 1 1 0 1 – 2 1

1 2 3 3 0 1 0 2 0 0 – 1 – 1 0 0 – 2 – 1

1 2 3 3 0 1 0 2 0 0 – 1 – 1 0 0 0 1 ⇒ ⇒ R a n k ( A ) = 4

Следовательно, система заданных векторов линейно независима и их численность равна размерности векторного пространства – они являются базисом четырехмерного векторного пространства.

Ответ: заданные векторы являются базисом четырехмерного пространства.

Исходные данные: векторы

a ( 1 ) = ( 1 , 2 , – 1 , – 2 ) a ( 2 ) = ( 0 , 2 , 1 , – 3 ) a ( 3 ) = ( 1 , 0 , 0 , 5 )

Составляют ли они базис пространства размерностью 4?

Решение

Исходная система векторов линейно независима, но численность векторов в ней недостаточна, чтобы стать базисом четырехмерного пространства.

Ответ: нет, не составляют.

Разложение вектора по базису

Примем, что произвольные векторы e ( 1 ) , e ( 2 ) , . . . , e ( n ) являются базисом векторного n-мерного пространства. Добавим к ним некий n -мерный вектор x → : полученная система векторов станет линейно зависимой. Свойства линейной зависимости гласят, что хотя бы один из векторов такой системы может линейно выражаться через остальные. Переформулируя это утверждение, можно говорить о том, что хотя бы один из векторов линейно зависимой системы может раскладываться по остальным векторам.

Таким образом, мы пришли к формулировке важнейшей теоремы:

Любой вектор n -мерного векторного пространства единственным образом раскладывается по базису.

Докажем эту теорему:

зададим базис n -мерного векторного пространства – e ( 1 ) , e ( 2 ) , . . . , e ( n ) . Сделаем систему линейно зависимой, добавив к ней n -мерный вектор x → . Этот вектор может быть линейно выражен через исходные векторы e :

x = x 1 · e ( 1 ) + x 2 · e ( 2 ) + . . . + x n · e ( n ) , где x 1 , x 2 , . . . , x n – некоторые числа.

Теперь докажем, что такое разложение является единственным. Предположим, что это не так и существует еще одно подобное разложение:

Отнимем от левой и правой частей этого равенства соответственно левую и правую части равенства x = x 1 · e ( 1 ) + x 2 · e ( 2 ) + . . . + x n · e ( n ) . Получим:

1 – x 1 ) · e ( 1 ) + ( x

2 – x 2 ) · e ( 2 ) + . . . ( x

Система базисных векторов e ( 1 ) , e ( 2 ) , . . . , e ( n ) линейно независима; по определению линейной независимости системы векторов равенство выше возможно только тогда, когда все коэффициенты ( x

2 – x 2 ) , . . . , ( x

n – x n ) будут равны нулю. Из чего справедливым будет: x 1 = x

n . И это доказывает единственный вариант разложения вектора по базису.

При этом коэффициенты x 1 , x 2 , . . . , x n называются координатами вектора x → в базисе e ( 1 ) , e ( 2 ) , . . . , e ( n ) .

Доказанная теория делает понятным выражение «задан n -мерный вектор x = ( x 1 , x 2 , . . . , x n ) »: рассматривается вектор x → n -мерного векторного пространства, и его координаты заданы в некотором базисе. При этом также понятно, что этот же вектор в другом базисе n -мерного пространства будет иметь другие координаты.

Рассмотрим следующий пример: допустим, что в некотором базисе n -мерного векторного пространства задана система из n линейно независимых векторов

e ( 1 ) = ( e 1 ( 1 ) , e 2 ( 1 ) , . . . , e n ( 1 ) ) e ( 2 ) = ( e 1 ( 2 ) , e 2 ( 2 ) , . . . , e n ( 2 ) ) ⋮ e ( n ) = ( e 1 ( n ) , e 2 ( n ) , . . . , e n ( n ) )

а также задан вектор x = ( x 1 , x 2 , . . . , x n ) .

Векторы e 1 ( 1 ) , e 2 ( 2 ) , . . . , e n ( n ) в этом случае также являются базисом этого векторного пространства.

Предположим, что необходимо определить координаты вектора x → в базисе e 1 ( 1 ) , e 2 ( 2 ) , . . . , e n ( n ) , обозначаемые как x

Вектор x → будет представлен следующим образом:

2 · e ( 2 ) + . . . + x

Запишем это выражение в координатной форме:

( x 1 , x 2 , . . . , x n ) = x

1 · ( e ( 1 ) 1 , e ( 1 ) 2 , . . . , e ( 1 ) n ) + x

2 · ( e ( 2 ) 1 , e ( 2 ) 2 , . . . , e ( 2 ) n ) + . . . + + x

n · ( e ( n ) 1 , e ( n ) 2 , . . . , e ( n ) n ) = = ( x

2 e 1 ( 2 ) + . . . + x

2 e 2 ( 2 ) + + . . . + x

n e 2 ( n ) , . . . , x

2 e n ( 2 ) + . . . + x

Полученное равенство равносильно системе из n линейных алгебраических выражений с n неизвестными линейными переменными x

n e 2 n ⋮ x n = x

Матрица этой системы будет иметь следующий вид:

e 1 ( 1 ) e 1 ( 2 ) ⋯ e 1 ( n ) e 2 ( 1 ) e 2 ( 2 ) ⋯ e 2 ( n ) ⋮ ⋮ ⋮ ⋮ e n ( 1 ) e n ( 2 ) ⋯ e n ( n )

Пусть это будет матрица A , и ее столбцы – векторы линейно независимой системы векторов e 1 ( 1 ) , e 2 ( 2 ) , . . . , e n ( n ) . Ранг матрицы – n , и ее определитель отличен от нуля. Это свидетельствует о том, что система уравнений имеет единственное решение, определяемое любым удобным способом: к примеру, методом Крамера или матричным методом. Таким образом мы сможем определить координаты x

n вектора x → в базисе e 1 ( 1 ) , e 2 ( 2 ) , . . . , e n ( n ) .

Применим рассмотренную теорию на конкретном примере.

Исходные данные: в базисе трехмерного пространства заданы векторы

e ( 1 ) = ( 1 , – 1 , 1 ) e ( 2 ) = ( 3 , 2 , – 5 ) e ( 3 ) = ( 2 , 1 , – 3 ) x = ( 6 , 2 , – 7 )

Необходимо подтвердить факт, что система векторов e ( 1 ) , e ( 2 ) , e ( 3 ) также служит базисом заданного пространства, а также определить координаты вектора х в заданном базисе.

Решение

Система векторов e ( 1 ) , e ( 2 ) , e ( 3 ) будет являться базисом трехмерного пространства, если она линейно независима. Выясним эту возможность, определив ранг матрицы A , строки которой – заданные векторы e ( 1 ) , e ( 2 ) , e ( 3 ) .

Используем метод Гаусса:

A = 1 – 1 1 3 2 – 5 2 1 – 3

1 – 1 1 0 5 – 8 0 3 – 5

1 – 1 1 0 5 – 8 0 0 – 1 5

R a n k ( A ) = 3 . Таким образом, система векторов e ( 1 ) , e ( 2 ) , e ( 3 ) линейно независима и является базисом.

Пусть в базисе вектор x → имеет координаты x

3 . Связь этих координат определяется уравнением:

3 e 1 ( 3 ) x 2 = x

3 e 2 ( 3 ) x 3 = x

Применим значения согласно условиям задачи:

Решим систему уравнений методом Крамера:

∆ = 1 3 2 – 1 2 1 1 – 5 – 3 = – 1 ∆ x

1 = 6 3 2 2 2 1 – 7 – 5 – 3 = – 1 , x

1 ∆ = – 1 – 1 = 1 ∆ x

2 = 1 6 2 – 1 2 1 1 – 7 – 3 = – 1 , x

2 ∆ = – 1 – 1 = 1 ∆ x

3 = 1 3 6 – 1 2 2 1 – 5 – 7 = – 1 , x

Так, вектор x → в базисе e ( 1 ) , e ( 2 ) , e ( 3 ) имеет координаты x

Ответ: x = ( 1 , 1 , 1 )

Связь между базисами

Предположим, что в некотором базисе n-мерного векторного пространства даны две линейно независимые системы векторов:

c ( 1 ) = ( c 1 ( 1 ) , c 2 ( 1 ) , . . . , c n ( 1 ) ) c ( 2 ) = ( c 1 ( 2 ) , c 2 ( 2 ) , . . . , c n ( 2 ) ) ⋮ c ( n ) = ( c 1 ( n ) , e 2 ( n ) , . . . , c n ( n ) )

e ( 1 ) = ( e 1 ( 1 ) , e 2 ( 1 ) , . . . , e n ( 1 ) ) e ( 2 ) = ( e 1 ( 2 ) , e 2 ( 2 ) , . . . , e n ( 2 ) ) ⋮ e ( n ) = ( e 1 ( n ) , e 2 ( n ) , . . . , e n ( n ) )

Указанные системы являются также базисами заданного пространства.

n ( 1 ) – координаты вектора c ( 1 ) в базисе e ( 1 ) , e ( 2 ) , . . . , e ( 3 ) , тогда связь координат будет задаваться системой линейных уравнений:

1 ( 1 ) e 1 ( 1 ) + c

2 ( 1 ) e 1 ( 2 ) + . . . + c

n ( 1 ) e 1 ( n ) с 2 ( 1 ) = c

1 ( 1 ) e 2 ( 1 ) + c

2 ( 1 ) e 2 ( 2 ) + . . . + c

n ( 1 ) e 2 ( n ) ⋮ с n ( 1 ) = c

1 ( 1 ) e n ( 1 ) + c

2 ( 1 ) e n ( 2 ) + . . . + c

В виде матрицы систему можно отобразить так:

( c 1 ( 1 ) , c 2 ( 1 ) , . . . , c n ( 1 ) ) = ( c

n ( 1 ) ) · e 1 ( 1 ) e 2 ( 1 ) … e n ( 1 ) e 1 ( 2 ) e 2 ( 2 ) … e n ( 2 ) ⋮ ⋮ ⋮ ⋮ e 1 ( n ) e 2 ( n ) … e n ( n )

Сделаем по аналогии такую же запись для вектора c ( 2 ) :

( c 1 ( 2 ) , c 2 ( 2 ) , . . . , c n ( 2 ) ) = ( c

n ( 2 ) ) · e 1 ( 1 ) e 2 ( 1 ) … e n ( 1 ) e 1 ( 2 ) e 2 ( 2 ) … e n ( 2 ) ⋮ ⋮ ⋮ ⋮ e 1 ( n ) e 2 ( n ) … e n ( n )

И, далее действуя по тому же принципу, получаем:

( c 1 ( n ) , c 2 ( n ) , . . . , c n ( n ) ) = ( c

n ( n ) ) · e 1 ( 1 ) e 2 ( 1 ) … e n ( 1 ) e 1 ( 2 ) e 2 ( 2 ) … e n ( 2 ) ⋮ ⋮ ⋮ ⋮ e 1 ( n ) e 2 ( n ) … e n ( n )

Матричные равенства объединим в одно выражение:

c 1 ( 1 ) c 2 ( 1 ) ⋯ c n ( 1 ) c 1 ( 2 ) c 2 ( 2 ) ⋯ c n ( 2 ) ⋮ ⋮ ⋮ ⋮ c 1 ( n ) c 2 ( n ) ⋯ c n ( n ) = c

n ( n ) · e 1 ( 1 ) e 2 ( 1 ) ⋯ e n ( 1 ) e 1 ( 2 ) e 2 ( 2 ) ⋯ e n ( 2 ) ⋮ ⋮ ⋮ ⋮ e 1 ( n ) e 2 ( n ) ⋯ e n ( n )

Оно и будет определять связь векторов двух различных базисов.

Используя тот же принцип, возможно выразить все векторы базиса e ( 1 ) , e ( 2 ) , . . . , e ( 3 ) через базис c ( 1 ) , c ( 2 ) , . . . , c ( n ) :

e 1 ( 1 ) e 2 ( 1 ) ⋯ e n ( 1 ) e 1 ( 2 ) e 2 ( 2 ) ⋯ e n ( 2 ) ⋮ ⋮ ⋮ ⋮ e 1 ( n ) e 2 ( n ) ⋯ e n ( n ) = e

n ( n ) · c 1 ( 1 ) c 2 ( 1 ) ⋯ c n ( 1 ) c 1 ( 2 ) c 2 ( 2 ) ⋯ c n ( 2 ) ⋮ ⋮ ⋮ ⋮ c 1 ( n ) c 2 ( n ) ⋯ c n ( n )

Дадим следующие определения:

n ( n ) является матрицей перехода от базиса e ( 1 ) , e ( 2 ) , . . . , e ( 3 )

к базису c ( 1 ) , c ( 2 ) , . . . , c ( n ) .

n ( n ) является матрицей перехода от базиса c ( 1 ) , c ( 2 ) , . . . , c ( n )

к базису e ( 1 ) , e ( 2 ) , . . . , e ( 3 ) .

Алгоритм нахождения базиса системы векторов

Для того чтобы найти базис системы векторов Av А2. А , необходимо:

1) составить соответствующую системе векторов однородную систему уравнений

2) привести эту систему к равносильной разрешенной системе вида

  • 3) записать базис системы векторов Б = (АрА2, . А ), включив в него векторы, соответствующие разрешенным неизвестным;
  • 4) записать разложения векторов по базису; коэффициентами разложения вектора А. по этому базису являются координаты соответствующего вектора

в разрешенной системе уравнений, т.е.

Система векторов, состоящая из п векторов, ранг которой равен г, может иметь несколько базисов. Число возможных базисов системы векторов определяется как число меньшее или равное числу сочетаний из п по г.

Пример 3.3. Найти ранг и базис системы векторов

разложения векторов по базису, перейти к новому базису и найти число возможных базисов системы.

Решение. Составим систему уравнений A t ay + А2х2 + . + А„хп = 0, которая в координатной записи имеет вид

Приведение данной системы уравнений с помощью преобразований Жордана к равносильной разрешенной приведено в ниже следующей таблице.

Разрешенная система имеет вид

В базис системы векторов включаем 1-й и 2-й векторы Б: = (AVA2), которые соответствуют разрешенным неизвестным х1 и х2. Ранг системы векторов равен числу векторов, вошедших в базис, т.е. г = 2.

Запишем разложения векторов по базису. Коэффициентами разложения вектора А3 являются координаты вектора А’3 = (3, -2), т.е. коэффициенты при х3 в разрешенной системе уравнений (в последних трех строках таблицы), они образуют столбец, расположенный под х3 А3 = ЗЛ1 – 2Аг Аналогично, коэффициентами разложения вектора А4 являются координаты вектора А’4 = (4, 1) А4 = 4Ау + 1 Ат

Для нахождения нового базиса необходимо выбрать новый разрешающий элемент. Пусть этим элементом будет элемент я94 = 1.

Векторные пространства

При проведении научных и прикладных исследование часто создаются модели, в которых рассматриваются точки и/или векторы определенных пространств. Например, в моделях шифров на эллиптических кривых используются аффинные и проективные пространства. К проективным прибегают тогда, когда необходимо ускорить вычисления, так как в формулах манипулирования с точками эллиптической кривой выводимых в рамках проективного пространства отсутствует операция деления на координату, которую в случае аффинного пространства обойти не удается.

Операция деления как раз одна из самых «дорогих» операций. Дело в том, что в алгебраических полях, а соответственно и в группах операция деления вообще отсутствует и выход из положения (когда не делить нельзя) состоит в том, что операцию деления заменяют умножением, но умножают не на саму координату, а на обращенное ее значение. Из этого следует, что предварительно надо привлекать расширенный алгоритм Евклида НОД и кое что еще. Одним словом, не все так просто как изображают авторы большинства публикаций о ЕСС. Почти все, что по этой теме опубликовано и не только в Интернете мне знакомо. Мало того, что авторы не компетентны и занимаются профанацией, оценщики этих публикаций плюсуют авторов в комментариях, т. е. не видят ни пробелов, ни явных ошибок. Про нормальную же статью пишут, что она уже 100500-я и от нее нулевой эффект. Так все пока на Хабре устроено, анализ публикаций делается огромный, но не качества содержания. Здесь возразить нечего — реклама двигатель бизнеса.

Линейное векторное пространство

Изучение и описание явлений окружающего мира с необходимостью приводит нас к введению и использованию ряда понятий таких как точки, числа, пространства, прямые линии, плоскости, системы координат, векторы, множества и др.

Пусть r = вектор трехмерного пространства, задает положение одной частицы (точки) относительно начала координат. Если рассматривать N элементов, то описание их положения требует задания 3∙N координат, которые можно рассматривать как координаты некоторого вектора в 3N-мерном пространстве. Если рассматривать непрерывные функции и их совокупности, то приходим к пространствам, размерность которых равна бесконечности. На практике часто ограничиваются использованием лишь подпространства такого бесконечномерного пространства функции координат, обладающего конечным числом измерений.

Пример 1. Ряд Фурье — пример использования пространства функций. Рассмотрим разложение произвольной функции в ряд Фурье

Его можно трактовать как разложение «вектора» f(x) по бесконечному набору «ортогональных» базисных векторов sinпх

Это пример абстрагирования и распространения понятия вектора на бесконечное число измерений. Действительно, известно, что при -π≤x≤π

Существо дальнейшего рассмотрения не пострадает, если мы отвлечемся от размерности абстрактного векторного пространства – будь — то 3, 3N или бесконечность, хотя для практических приложений больший интерес представляет конечномерные поля и векторные пространства.

Набор векторов r1, r2,… будем называть линейным векторным пространством L, если сумма любых двух его элементов тоже находится в этом наборе и если результат умножения элемента на число С также входит в этот набор. Оговоримся сразу, что значения числа С могут быть выбраны из вполне определенного числового множества Fр – поля вычетов по модулю простого числа р, которое считается присоединенным к L.

Пример 2. Набор из 8 векторов, составленных из n =5 -разрядных двоичных чисел
r0 = 00000, r1 = 10101, r2 = 01111, r3 = 11010, r4 = 00101, r5 = 10110, r6 = 01001, r7 = 11100 образует векторное пространство L, если числа С є <0,1>. Этот небольшой пример позволяет убедиться в проявлении свойств векторного пространства, включенных в его определение.

Суммирование этих векторов выполняется поразрядно по модулю два, т. е. без переноса единиц в старший разряд. Отметим, что если все С действительные (в общем случае С принадлежат полю комплексных чисел), то векторное пространство называют действительным.

Формально аксиомы векторного пространства и записываются так:
r1 + r2 = r2 + r1 = r3; r1, r2, r3 є L – коммутативность сложения и замкнутость;
(r1 + r2) + r3 = r1 + (r2 + r3) = r1 + r2 + r3 – ассоциативность сложения;
ri + r0 = r0 + ri = ri; ∀i, ri, r0 є L–существование нейтрального элемента;
ri +(- ri) = r0, для ∀i существует противоположный вектор (-ri) є L;
1∙ ri = ri ∙1 = ri существование единицы для умножения;
α (β∙ri) = (α∙β)∙ri; α, β, 1, 0 – элементы числового поля F, ri є L; умножение на скаляры ассоциативно; результат умножения принадлежит L;
(α + β) ri = α∙ri + β∙ri; для ∀i, ri є L, α, β – скаляры;
а (ri + rj) = ari + arj для всех а, ri, rj є L;
a∙0 = 0, 0∙ri = 0; (-1) ∙ ri = – ri.

Размерность и базис векторного пространства

При изучении векторных пространств представляет интерес выяснение таких вопросов, как число векторов, образующих все пространство; какова размерность пространства; какой наименьший набор векторов путем применения к нему операции суммирования и умножения на число позволяет сформировать все векторы пространства? Эти вопросы основополагающие и их нельзя обойти стороной, так как без ответов на них утрачивается ясность восприятия всего остального, что составляет теорию векторных пространств.

Оказалось, что размерность пространства самым тесным образом связана с линейной зависимостью векторов, и с числом линейно независимых векторов, которые можно выбирать в изучаемом пространстве многими способами.

Линейная независимость векторов

Набор векторов r1, r2, r3 … rр из L называют линейно независимым, если для них соотношение

выполняется только при условии одновременного равенства .
Все , k = 1(1)p, принадлежат числовому полю вычетов по модулю два
F = <0, 1>.
Если в некотором векторном пространстве L можно подобрать набор из р векторов, для которых соотношение выполняется, при условии, что не все одновременно, т.е. в поле вычетов оказалось возможным выбрать набор , k =1(1)р, среди которых есть ненулевые, то такие векторы называются линейно зависимыми.

Пример 3. На плоскости два вектора = T и = T являются линейно независимыми, так как в соотношении (T-транспонирование)

невозможно подобрать никакой пары чисел коэффициентов не равных нулю одновременно, чтобы соотношение было выполнено.
Три вектора = T , = T , = T образуют систему линейно зависимых векторов, так как в соотношении

равенство может быть обеспечено выбором коэффициентов , не равных нулю одновременно. Более того, вектор является функцией и (их суммой), что указывает на зависимость от и . Доказательство общего случая состоит в следующем.

Пусть хотя бы одно из значений , k = 1(1)р, например, , а соотношение выполнено. Это означает, что векторы , k = 1(1)р, линейно зависимы

Выделим явным образом из суммы вектор rр

Говорят, что вектор rр является л и н е й н о й комбинацией векторов или rр через остальные векторы выражается линейным образом, т.е. rр линейно зависит от остальных. Он является их функцией.

На плоскости двух измерений любые три вектора линейно зависимы, но любые два неколлинеарных вектора являются независимыми. В трехмерном пространстве любые три некомпланарных вектора линейно независимы, но любые четыре вектора всегда линейно зависимы.

Зависимость/независимость совокупности <> векторов часто определяют, вычисляя определитель матрицы Грама (ее строки скалярные произведения наших векторов). Если определитель равен нулю, среди векторов имеются зависимые, если определитель отличен от нуля — векторы в матрице независимы.

Определителем Грама (грамианом) системы векторов

в евклидовом пространстве называется определитель матрицы Грама этой системы:

где — скалярное произведение векторов
и .

Размерность и базис векторного пространства

Размерность s = d (L) пространства L определяется как наибольшее число векторов в L, образующих линейно независимый набор. Размерность – это не число векторов в L, которое может быть бесконечным и не число компонентов вектора.

Пространства, имеющие конечную размерность s ≠ ∞, называются конечномерными, если
s = ∞, – бесконечномерными.

Ответом на вопрос о минимальном числе и составе векторов, которые обеспечивают порождение всех векторов линейного векторного пространства является следующее утверждение.

Любой набор s линейно независимых векторов в пространстве L образует его б а з и с. Это следует из того, что любой вектор линейного s-мерного векторного пространства L может быть представлен единственным способом в виде линейной комбинации векторов базиса.

Зафиксируем и обозначим символом , i = 1(1)s, один из наборов, образующих базис пространства L. Тогда

Числа rki, i = 1(1)s называются координатами вектора в базисе , i = 1(1)s, причем rki = (, ).
Покажем единственность представления . Очевидно, что набор , является зависимым, так как , i = 1(1)s – базис. Другими словами, существуют такие не равные одновременно нулю, что .
При этом пусть , ибо если , то хоть одно из , было бы отлично от нуля и тогда векторы , i = 1(1)s, были бы линейно зависимы, что невозможно, так как это базис. Следовательно,

, будем иметь
Используя прием доказательства «от противного», допустим, что записанное представление не единственное в этом базисе и существует другое

Тогда запишем отличие представлений, что, естественно, выражается как

Очевидно, что правая и левая части равны, но левая представляет разность вектора с самим собой, т. е. равна нулю. Следовательно, и правая часть равна нулю. Векторы , i = 1(1)s линейно независимы, поэтому все коэффициенты при них могут быть только нулевыми. Отсюда получаем, что

а это возможно только при

Выбор базиса. Ортонормированность

Векторы называют нормированными, если длина каждого из них равна единице. Этого можно достичь, применяя к произвольным векторам процедуру нормировки.

Векторы называют ортогональными, если они перпендикулярны друг другу. Такие векторы могут быть получены применением к каждому из них процедуры ортогонализации. Если для совокупности векторов выполняются оба свойства, то векторы называются ортонормированными.

Необходимость рассмотрения ортонормированных базисов вызвана потребностями использования быстрых преобразований как одно –, так и многомерных функций. Задачи такой обработки возникают при исследовании кодов, кодирующих информационные сообщения в сетях связи различного назначения, при исследовании изображений, получаемых
посредством автоматических и автоматизированных устройств, в ряде других областей, использующих цифровые представления информации.

Определение. Совокупность n линейно независимых векторов n-мерного векторного
пространства V называется его базисом.

Теорема. Каждый вектор х линейного n-мерного векторного пространства V можно представить, притом единственным образом, в виде линейной комбинации векторов базиса. Векторное пространство V над полем F обладает следующими свойствами:
0·х = 0 (0 в левой части равенства – нейтральный элемент аддитивной группы поля F; 0 в правой части равенства – элемент пространства V, являющийся нейтральным единичным элементом аддитивной группы V, называемый нулевым вектором);
(– 1)·х = –х; –1є F; x є V; –x є V;
Если α·х = 0єV, то при х ≠ 0 всегда α = 0.
Пусть Vn(F) – множество всех последовательностей (х1, х2, …, хn) длины n с компонентами из поля F, т.е. Vn(F) = i =1(1)n >.

Сложение и умножение на скаляр определяются следующим образом:
x + y =(x1 + y1, x2 + y2, …, xn + yn);
α·х = (α·х1, α·х2,…, α·хn), где у = (у1, у2,…, уn),
тогда Vn(F) является векторным пространством над полем F.

Пример 4. В векторном пространстве rо = 00000, r1 = 10101, r2 = 11010, r3 = 10101 над полем F2 = <0,1>определить его размерность и базис.
Решение. Сформируем таблицу сложения векторов линейного векторного пространства

В этом векторном пространстве V= каждый вектор в качестве противоположного имеет самого себя. Любые два вектора, исключая rо, являются линейно независимыми, в чем легко убедиться
c1·r1 + c2·r2 = 0; c1·r1 + c3·r3 = 0; c2·r2 + c3·r3 = 0;

Каждое из трех соотношений справедливо только при одновременных нулевых значениях пар коэффициентов сi, сj є <0,1>.

При одновременном рассмотрении трех ненулевых векторов один из них всегда является суммой двух других или равен самому себе, а r1+r2+r3=rо.

Таким образом, размерность рассматриваемого линейного векторного пространства равна двум s = 2, d(L) = s = 2, хотя каждый из векторов имеет пять компонентов. Базисом пространства является набор (r1, r2). Можно в качестве базиса использовать пару (r1, r3).

Важным в теоретическом и практическом отношении является вопрос описания векторного пространства. Оказывается, любое множество базисных векторов можно рассматривать как строки некоторой матрицы G, называемой порождающей матрицей векторного пространства. Любой вектор этого пространства может быть представлен как линейная комбинация строк матрицы G ( как, например, здесь).

Если размерность векторного пространства равна k и равна числу строк матрицы G, рангу матрицы G, то очевидно, существует k коэффициентов с q различными значениями для порождения всех возможных линейных комбинаций строк матрицы. При этом векторное пространство L содержит q k векторов.

Множество всех векторов из ℤpn с операциями сложения векторов и умножения вектора на скаляр из ℤp есть линейное векторное пространство.

Определение. Подмножество W векторного пространства V, удовлетворяющее условиям:
Если w1, w2 є W, то w1+ w2 є W,
Для любых α є F и w є W элемент αw є W,
само является векторным пространством над полем F и называется подпространством векторного пространства V.

Пусть V есть векторное пространство над полем F и множество W ⊆ V. Множество W есть подпространство пространства V, если W по отношению к линейным операциям, определенным в V, есть линейное векторное пространство.

Таблица. Характеристики векторных пространств

Компактность матричного представления векторного пространства очевидна. Например, задание L векторов двоичных 50-разрядных чисел, среди которых 30 векторов образуют базис векторного пространства, требует формирования матрицы G[30,50], а описываемое количество векторов превышает 10 9 , что в поэлементной записи представляется неразумным.

Все базисы любого пространства L разбиваются подгруппой Р невырожденных матриц с det G > 0 на два класса. Один из них (произвольно) называют классом с положительно ориентированными базисами (правыми), другой класс содержит левые базисы.

В этом случае говорят, что в пространстве задана ориентация. После этого любой базис представляет собой упорядоченный набор векторов.

Если нумерацию двух векторов изменить в правом базисе, то базис станет левым. Это связано с тем, что в матрице G поменяются местами две строки, следовательно, определитель detG изменит знак.

Норма и скалярное произведение векторов

После того как решены вопросы о нахождении базиса линейного векторного пространства, о порождении всех элементов этого пространства и о представлении любого элемента и самого векторного пространства через базисные векторы, можно поставить задачу об измерении в этом пространстве расстояний между элементами, углов между векторами, значений компонентов векторов, длины самих векторов.

Действительное или комплексное векторное пространство L называется нормированным векторным пространством, если каждый вектор r в нем может быть сопоставлен действительному числу || r || – модулю вектора, норме. Единичный вектор – это вектор, норма которого равна единице. Нулевой вектор имеет компонентами нули.

Определение. Векторное пространство называется унитарным, если в нем определена бинарная операция, ставящая каждой паре ri, rj векторов из L в соответствие скаляр. В круглых скобках (ri, rj) записывается (обозначается) скалярное или внутреннее произведение ri и rj, причем
1. (ri, rj) = ri ∙ rj;
2. (ri, rj) = (rj ∙ ri)*, где * указывает на комплексное сопряжение или эрмитову симметрию;
3. (сri, rj) = с(ri ∙ rj) – ассоциативный закон;
4. (ri + rj, rk) = (ri ∙ rk)+ (rj ∙ rk)– дистрибутивный закон;
5. (ri, rk) ≥ 0 и из (ri, rj ) = 0 следует ri = 0.

Определение. Положительное значение квадратного корня называют нормой (или длиной, модулем) вектора ri. Если = 1, то вектор ri называют нормированным.

Два вектора ri, rj унитарного векторного пространства L взаимно ортогональны, если их скалярное произведение равно нулю, т.е. (ri, rj) = 0.

При s = 3 в линейном векторном пространстве в качестве базиса удобно выбирать три взаимно перпендикулярных вектора. Такой выбор существенно упрощает ряд зависимостей и вычислений. Этот же принцип ортогональности используется при выборе базиса в пространствах и других размерностей s > 3. Использование введенной операции скалярного произведения векторов обеспечивает возможность такого выбора.

Еще большие преимущества достигаются при выборе в качестве базиса векторного пространства ортогональных нормированных векторов – ортонормированного базиса. Если не оговорено специально, то далее всегда будем считать, что базис еi, i = 1(1)s выбран именно таким образом, т.е.

, где ij — символ Кронекера (1823 — 1891).

В унитарных векторных пространствах такой выбор всегда реализуем. Покажем реализуемость такого выбора.

Определение. Пусть S = есть конечное подмножество векторного пространства V над полем F.
Линейная комбинация векторов из S есть выражение вида а1∙v1 + а2∙v2 +…+ аn∙vn, где каждое аi ∊ F.

Оболочка для множества S (обозначение ) есть множество всех линейных комбинаций векторов из S. Оболочка для S есть подпространство пространства V.

Если U есть пространство в V, то U натянуто на S (S стягивает U), если =U.
Множество векторов S линейно зависимо над F, если в F существуют скаляры а1, а2,…, аn, не все нули, для которых а1∙v1+ а2∙v2 +…+ аn∙vn = 0. Если таких скаляров не существует, то множество векторов S линейно независимо над F.

Если векторное пространство V натянуто на линейно независимую систему векторов S (или система S стягивает пространство V), то система S называется базисом для V.

Приведение произвольного базиса к ортонормированному виду

Известно следующее утверждение [11]. Если ē i, i = 1(1)s – произвольная конечная или счетная система линейно независимых векторов в унитарном векторном пространстве, то существует ортонормированная система ē i, i = 1(1)s, порождающая то же самое линейное пространство (многообразие).

В основу процедуры приведения базиса к ортонормированному виду положен процесс ортогонализации Грама — Шмидта, который в свою очередь, реализуется рекуррентными формулами

В развернутом виде алгоритм ортогонализации и нормирования базиса содержит следующие условия:

Делим вектор ē 1, на его норму; получим нормированный вектор ē i1/(||ē 1 ||);
Формируем V2 = ē 2 — (ē 1, ē 2)e 1 и нормируем его, получим е 2. Ясно, что тогда
(е1, е2)

(е1, е2) – (е1, ē 2)( е1, е1) = 0;
Построив V3 = ē 3– (e1, ē 3)e1 – (e2, ē 3) e2 и нормируя его, получим е3.

Для него имеем сразу же (е1, е3) = (е2, е3) = 0.
Продолжая такой процесс, получим ортонормированный набор ē i, i = 1(1)s. Этот набор содержит линейно независимые векторы, поскольку все они взаимно ортогональны.
Убедимся в этом. Пусть выполняется соотношение

Если набор ē i, i = 1(1)s зависимый, то хотя бы один сj коэффициент не равен нулю сj ≠ 0.

Умножив обе части соотношения на еj, получаем
(ej, c1∙e1 ) + (ej, c2∙e2 )+ . + ( ej, cj∙ej ) +…+ ( ej, cs∙rs ) = 0.
Каждое слагаемое в сумме равно нулю как скалярное произведение ортогональных векторов, кроме (ej ,cj∙ej), которое равно нулю по условию. Но в этом слагаемом
(ej, ej) = 1 ≠ 0, следовательно, нулем может быть только cj.
Таким образом, допущение о том, что cj ≠ 0 неверно и набор является линейно независимым.

Пример 5. Задан базис 3-х мерного векторного пространства:
< , , >.
Скалярное произведение определено соотношением:
( , ) = x1∙y1+x2∙y2+x3∙y3+x4∙y4.
Процедурой ортогонализации Грама — Шмидта получаем систему векторов:
а1 = ; a2 = -4 /7= /7;
a3 = +½ — /5 = /10.
(a1,a2)= (1+4+9+0) = 14;
a1 E =a1/√14;
a2-(a1 E ,a2)∙a1 E =a2-(8/√14)(a1/√14)=a2 — 4∙a1/7;
Третий вектор читателю предлагается обработать самостоятельно.

Нормированные векторы получают вид:
a1 E =a1/√14;
a2 E = /√70;
a3 E = /√70;

Ниже в примере 6 дается подробный развернутый процесс вычислений получения ортонормированного базиса из простого (взятого наугад).

Пример 6. Привести заданный базис линейного векторного пространства к ортонормированному виду.
Дано: векторы базиса

Подпространства векторных пространств

Структура векторного пространства

Представление объектов (тел) в многомерных пространствах весьма непростая задача. Так, четырехмерный куб в качестве своих граней имеет обычные трехмерные кубы, и в трехмерном пространстве может быть построена развертка четырехмерного куба. В некоторой степени «образность» и наглядность объекта или его частей способствует более успешному его изучению.

Сказанное позволяет предположить, что векторные пространства можно некоторым образом расчленять, выделять в них части, называемые подпространствами. Очевидно, что рассмотрение многомерных и тем более бесконечномерных пространств и объектов в них лишает нас наглядности представлений, что весьма затрудняет исследование объектов в таких
пространствах. Даже, казалось бы, такие простые вопросы, как количественные характеристики элементов многогранников (число вершин, ребер, граней, и т. п.) в этих пространствах решены далеко не полностью.

Конструктивный путь изучения подобных объектов состоит в выделении их элементов (например, ребер, граней) и описании их в пространствах меньшей размерности. Так четырехмерный куб в качестве своих граней имеет обычные трехмерные кубы и в трехмерном пространстве может быть построена развертка четырехмерного куба. В некоторой степени
«образность» и наглядность объекта или его частей способствует более успешному их изучению.

Если L – расширение поля К, то L можно рассматривать как векторное (или линейное) пространство над полем К. Элементы поля L (т. е. векторы) образуют по сложению абелеву группу. Кроме того, каждый «вектор» а є L может быть умножен на «скаляр» r є K, и при этом произведение ra снова принадлежит L (здесь ra – просто произведение в смысле операции поля L элементов r и а этого поля). Выполняются также законы
r∙(a+b) = r∙a+r∙b, (r+s)∙a = r∙a + r∙s, (r∙s)∙a = r∙(s∙a) и 1∙а = а, где r,s є K, a,b є L.

Сказанное позволяет предположить, что векторные пространства можно некоторым образом расчленять, выделять в них части, называемые подпространствами. Очевидно, что основным результатом при таком подходе является сокращение размерности выделяемых подпространств. Пусть в векторном линейном пространстве L выделены подпространства L1 и L2. В качестве базиса L1 выбирается меньший набор еi, i = 1(1)s1, s1 n – 1 способами. Следующий вектор v2 ≠ 0 не может быть выражен линейно через v1, т.е. может быть выбран q n – q способами и т.д.

Последний вектор vk ≠ 0 также линейно не выражается через предыдущие выбранные векторы v1,v2,…,vk и, следовательно, может быть выбран q n – q k – 1 способами. Общее число способов для выбора совокупности векторов v1,v2,…,vk, таким образом, определится как произведение числа выборов отдельных векторов, что и дает формулу (1). Для случая, когда k = п, имеем wп = wn, n и из формулы (I) получаем формулу (2).

Важные обобщающие результаты о размерностях подпространств.
Совокупность всех наборов длины n, ортогональных подпространству V1 наборов длины n, образует подпространство V2 наборов длины n. Это подпространство V2 называется нулевым пространством для V1.
Если вектор ортогонален каждому из векторов, порождающих подпространство V1, то этот вектор принадлежит нулевому пространству для V1.
Примером (V1) может служить множество 7-разрядных векторов порождающей матрицы (7,4)-кода Хемминга, с нулевым подпространством (V2) 7-разрядных векторов, образующих проверочную матрицу этого кода.

Если размерность подпространства (V1) наборов длины n равна k, то размерность нулевого подпространства (V2) равна n — k.

Если V2 — подпространство наборов длины n и V1 — нулевое пространство для V2, то (V2) — нулевое пространство для V1.

Пусть U∩V обозначает совокупность векторов, принадлежащих одновременно U и V, тогда U∩V является подпространством.

Пусть U⊕V обозначает подпространство, состоящее из совокупности всех линейных комбинаций вида au +bv, где u є U, v є V, a b — числа.

Сумма размерностей подпространств U∩V и U⊕V равна сумме размерностей подпространств U и V.

Пусть U2 — нулевое подпространство для U1, а V2 -нулевое пространство для V1. Тогда U2∩V2 является нулевым пространством для U1⊕V1.

Заключение

В работе рассмотрены основные понятия векторных пространств, которые часто используются при построении моделей анализа систем шифрования, кодирования и стеганографических, процессов, протекающих в них. Так в новом американском стандарте шифрования использованы пространства аффинные, а в цифровых подписях на эллиптических кривых и аффинные и
проективные (для ускорения обработки точек кривой).

Об этих пространствах в работе речь не идет (нельзя валить все в одну кучу, да и объем публикации я ограничиваю), но упоминания об этом сделаны не зря. Авторы, пишущие о средствах защиты, об алгоритмах шифров наивно полагают, что понимают детали описываемых явлений, но понимание евклидовых пространств и их свойств без всяких оговорок переносится в другие пространства, с другими свойствами и законами. Читающая аудитория вводится в заблуждение относительно простоты и доступности материала.

Создается ложная картина действительности в области информационной безопасности и специальной техники (технологий и математики).

В общем почин мною сделан, насколько удачно судить читателям.

[spoiler title=”источники:”]

http://bstudy.net/719717/estestvoznanie/algoritm_nahozhdeniya_bazisa_sistemy_vektorov

http://habr.com/ru/post/514806/

[/spoiler]

Определение.
Пусть К
– векторное пространство над полем Р;
предположим, что в этом пространстве
существует конечное число n
таких линейно независимых векторов
,
что всякий векторизК
линейно зависит от
.
Тогда будем говорить, что совокупностьобразуетбазис
пространства К
и, что векторное пространство К
имеет конечную размерность
n,
и записывается dimK
= n.

Замечание.
Существуют
векторные пространства, не имеющие
конечной размерности; говорят, что они
имеют бесконечную размерность; в таких
векторных пространствах имеются сколь
угодно большие совокупности линейно
независимых векторов. Например, векторное
пространство многочленов. Рассмотрение
таких пространств выходит за рамки
нашего курса линейной алгебры.

Не
существует базиса и в нулевом пространстве,
так как система, состоящая из одного
нулевого вектора, является линейно
зависимой. Размерность нулевого
пространства не определена и считается
равной нулю.

Следствия
из определения.

1.
В n
– мерном векторном пространстве К
совокупность, состоящая более чем из n
векторов, всегда линейно зависима.

  1. Если
    К
    имеет несколько базисов, то эти базисы
    содержат одинаковое

число
векторов, и число это равно размерности
К;
следовательно, dimK
не зависит от выбора базиса. Действительно,
если К
имеет базис, отличный от
,
последний будет иметьn
векторов, причем n

n.
Точно также в К
может существовать не более n
линейно независимых векторов, а значит
n  n‘,
и, следовательно, n
= n‘.

5.1. Построение базиса

Пусть
имеется n
– мерное векторное пространство К,
т.е. в нем существует хотя бы один базис
из n
векторов. Выберем в К
произвольный вектор
.
ЕслиК
не содержит векторов, линейно независимых
от
,
то для любого вектораимеемилиисоставляет
базис пространстваК,
которое имеет размерность 1.

Допустим,
что размерность n
> 1. Обозначим через
вектор
изК
линейно независимый от
.
Предположим, что таким путем постепенно
получены линейно независимые вектора.
Еслиr

n,
то К
содержит вектора линейно независимые
от
,
иначе эти вектора составляли бы базисК,
содержащим r

n =
dimK
векторов, что невозможно. Стало быть,
найдется такой вектор
,
что,
линейно независимы. Этим способом можно
получитьn
линейно
независимых векторов, которые и составят
базис пространства К.
Тот факт, что вектора для построения
базиса были выбраны произвольно,
свидетельствует о том, что всегда
существует бесконечное множество
различных базисов пространства К
(но все они содержат одинаковое число
векторов n
= dimK).
Тем самым можно считать доказанным
также теорему о неполном базисе и лемму
о замещении.

Теорема
о неполном базисе.

Всякую линейно независимую совокупность
векторов
гдеr

n
= dimK всегда
можно дополнить
n
r
другими векторами из К
так, чтобы полученная система n
векторов составляла базис пространства
К.

Лемма
о замещении.

Пусть
базис
пространстваК.
Тогда любой вектор
из этого базиса можно заменить другим
векторомизК,
который не является линейной комбинацией
остальных векторов в базисе:

.
Тогда– базисК.

Соседние файлы в предмете [НЕСОРТИРОВАННОЕ]

  • #
  • #
  • #
  • #
  • #
  • #
  • #
  • #
  • #
  • #
  • #

Все курсы > Линейная алгебра > Занятие 2

Продолжим работать в том же ноутбуке⧉

Определение

С понятием вектора тесно связано понятие векторного или линейного пространства (vector space, linear space).

По большому счету, векторное пространство — это множество векторов, которые мы можем складывать (vector addition) и умножать на число или скаляр (scalar multiplication).

В частности, сложение и умножение на число двумерных (состоящих из двух компонентов) векторов дает нам двумерный вектор, трехмерных — трехмерный и так далее.

$$ begin{bmatrix} 3 \ 4 end{bmatrix} + 2 cdot begin{bmatrix} 1 \ 2 end{bmatrix} = begin{bmatrix} 5 \ 8 end{bmatrix} $$

При этом сложить, например, двумерный и трехмерный вектор нельзя

$$ begin{bmatrix} 3 \ 4 end{bmatrix} + begin{bmatrix} 4 \ 3 \ 3 end{bmatrix} = ? $$

Также нельзя сформировать векторное пространство из двумерных векторов, лежащих только в первой четверти координатной плоскости. Хотя для таких векторов будет задана операция сложения, при умножении на отрицательный скаляр мы можем выйти за пределы первой четверти.

Поэтому говорят, что векторное пространство должно быть замкнуто относительно операций сложения и умножения на скаляр (closed under vector addition and scalar multiplication).

Двумерное пространтсво вещественных чисел принято обозначать $R^2$, трехмерное $R^3$, n-мерное — $R^n$.

Отметим, что вектор $begin{bmatrix} 1 \ 2 \ 0 end{bmatrix}$ — это вектор в $R^3$ с нулевым третьим компонентом.

Линейная комбинация векторов

Любой вектор внутри одного пространства (например, $R^2$) можно представить как линейную комбинацию конечного числа векторов (linear combination of a finite set of vectors).

$$ 2 cdot begin{bmatrix} 1 \ 2 end{bmatrix} + 3 cdot begin{bmatrix} 2 \ 1 end{bmatrix} = begin{bmatrix} 8 \ 7 end{bmatrix} $$

Под линейной комбинацией, как вы видите, понимается опять же сложение векторов и их умножение на число.

Аксиомы векторных пространств

Операции в векторных пространствах должны отвечать следующим правилам:

  1. $ mathbf u + (mathbf v + mathbf w) = (mathbf u + mathbf v) + mathbf w $
  2. $ mathbf v + mathbf w = mathbf w + mathbf v $
  3. Существует нулевой вектор $ mathbf 0 Rightarrow mathbf 0 + mathbf v = mathbf v, forall mathbf v $
  4. Для каждого $ mathbf v $ существует $ -mathbf v Rightarrow mathbf v + (-mathbf v) $
  5. $ a(b mathbf v) = (ab) mathbf v $
  6. $ 1 mathbf v = mathbf v $
  7. $ a(mathbf v + mathbf w) = a mathbf v + a mathbf w $
  8. $ (a + b) mathbf v = a mathbf v + b mathbf v $

Этим правилам могут отвечать не только векторы действительных чисел в пространстве $R^n$ (Евклидово пространство), но и, в частности, векторы функций. В этом случае речь идет о функциональных пространствах (function spaces).

Видео про абстрактные векторные пространства⧉.

Примечание. Некоторые понятия, упомянутые в видео выше, в частности, линейные преобразования (linear transformations), ядро матрицы (null space) и собственные векторы и значения (eigenvectors and eigenvalues) будут рассмотрены на более поздних занятиях.

Серия видео про алгебраические структуры⧉.

Внутреннее произведение

Скалярное произведение (dot product) является частным случаем внутреннего произведения (inner product) для евклидового пространства.

Приведем простой пример того, почему скалярное произведение может не подойти для векторов, состоящих, например, из комплексных чисел $mathbb C$. Ранее мы сказали, что скалярное произведение вектора самого на себя есть квадрат длины этого вектора, т.е. $mathbf x^T mathbf x = || mathbf x ||^2 $, и нам бы хотелось, чтобы квадрат длины был положителен.

Для действительных векторов это условие выполняется всегда, так как мы возводим в квадрат каждый компонент (вещественное число) такого вектора. Теперь рассмотрим комплексный вектор

$$ mathbf z = begin{bmatrix} 1 \ i end{bmatrix} in mathbb C^2 $$

По правилам скалярного произведения квадрат его длины был бы равен

$$ mathbf z^T mathbf z = begin{bmatrix} 1 & i end{bmatrix} begin{bmatrix} 1 \ i end{bmatrix} = 1 cdot 1 + i cdot i = 1-1 = 0 $$

Для того чтобы квадрат ненулевого вектора не был равен нулю в векторных пространствах комплексных чисел скалярное (а точнее внутреннее) произведение задано как $overline{mathbf z}^T mathbf z$, где $overline{mathbf z}$ является комплексно сопряженным (complex conjugate) к $mathbf z$ вектором. Тогда,

$$ overline{mathbf z}^T mathbf z = begin{bmatrix} 1 & -i end{bmatrix} begin{bmatrix} 1 \ i end{bmatrix} = 1 cdot 1 + (-i) cdot i = 1+1 = 2 $$

Векторное подпространство

Определение

Подпространством (subspace) векторного пространства $K$ называется множество $S$ его элементов, само являющееся векторным пространством относительно введенных в $K$ операций сложения и умножения на число.

Другими словами, чтобы $S$ было подпространством $K$ для каждого $mathbf v, mathbf w in S, K$ и $a in mathbb{R}$ должно выполняться ${ mathbf v + mathbf w, a mathbf v } in S, K$.

Например, выше мы сказали, что векторы $R^2$ первой четверти координатной плоскости не могут образовывать векторное пространство, потому что мы не можем задать для них операцию умножения на число, результатом которой был бы вектор только в этой четверти.

При этом, если мы возьмем на пространстве $R^2$ подпространство всех векторов, лежащих на прямой линии и проходящих через начало координат, то такое подпространство будет отвечать аксиомам векторных пространств.

подпространство R2

Примечание. $R^1$ нельзя назвать подпространством $R^2$, потому что у векторов $R^1$ только один компонент, а у векторов $R^2$, даже тех, которые лежат на одной линии, их два.

Пересечение подпространств

Если $S$ и $T$ — подпространства, то $S cap T$ тоже подпространство. Возьмем $mathbf v, mathbf w in S, T$. Тогда $mathbf v + mathbf w in S, T$ и $a mathbf v in S,T$, так как $S$ и $T$ отвечают свойствам подпространств.

Ортогональные подпространства

Подпространство $S$ будет ортогонально подпространству $T$, если каждый вектор в $S$ ортогонален каждому вектору в $T$.

$$ forall mathbf v in S perp forall mathbf w in T $$

Ортогональное дополнение

Если внутри некоторого пространства $K$ существует подпространство $S$, и внутри этого же пространства $K$ можно найти другое ортогональное ему подпространство $S^{perp}$, то такое подпространство называется ортогональным дополнением (orthogonal complement) подпространства $S$.

Более формально, пусть $S$ — подпространство $K$. Тогда ортогональным дополнением $S^{perp}$ будет множество всех векторов $mathbf w in K$, для которых скалярное произведение $mathbf w cdot mathbf v = 0$ для любого $mathbf v in S$.

$$ S^{perp} = { mathbf w in K hspace{5pt} | hspace{5pt} mathbf w cdot mathbf v = 0, hspace{5pt} forall mathbf v in S hspace{2pt} } $$

Покажем, что $S^{perp}$ также является подпространством. Возьмем два вектора $mathbf a, mathbf b in S^{perp} $. Для того чтобы $S^{perp}$ было подпространством, нам нужно продемонстрировать замкнутость относительно сложения и умножения на скаляр.

$$ mathbf a + mathbf b underset{mathord{?}}{in} S^{perp} $$

$$ c cdot mathbf a underset{mathord{?}}{in} S^{perp} $$

Начнем со сложения. По определению ортогонального дополнения

$$ mathbf a cdot mathbf v = 0, hspace{5pt} forall mathbf v in S $$

$$ mathbf b cdot mathbf v = 0, hspace{5pt} forall mathbf v in S $$

Тогда,

$$ (mathbf a + mathbf b) cdot mathbf v = mathbf a cdot mathbf v + mathbf b cdot mathbf v = mathbf 0 + mathbf 0 = mathbf 0 $$

Другими словами, мы показали, что сумма векторов принадлежащих $ S^{perp} $ также ортогонально вектору $mathbf v in S$, а значит принадлежит $ S^{perp} $, $ mathbf a + mathbf b in S^{perp} $. Перейдем к умножению на скаляр.

$$ c cdot mathbf a cdot mathbf v = c cdot ( mathbf a cdot mathbf v) = c cdot mathbf 0 = mathbf 0 $$

Это доказывает, что $ c cdot mathbf a in S^{perp} $.

Таким образом, ортогональное дополнение подпространства само является векторным подпространством.

Отметим, что нулевой вектор всегда принадлежит ортогональному дополнению, поскольку при $c = 0$

$$ c cdot mathbf a = 0 cdot mathbf a = mathbf 0 rightarrow mathbf 0 in S^{perp} $$

Обратите внимание, что нулевой вектор также присутствует в подпространстве $S$, $mathbf 0 in S$.

Более того, $S cap S^{perp} = { mathbf 0 } $. Это легко доказать. Возьмем некоторый вектор $mathbf x$, который одновременно принадлежит $ S $ и $ S^{perp} $, $mathbf x in S, S^{perp}$.

Тогда по определению ортогонального дополнения должно выполняться $mathbf x cdot mathbf x = 0 $. Такому условию отвечает только нулевой вектор.

Линейная независимость векторов

Когда один вектор можно выразить через умножение другого вектора на число говорят, что эти векторы линейно зависимы (linearly dependent). С двумя линейно независимыми (linearly independent) векторами $ mathbf v_1, mathbf v_2 $ такого сделать не получится.

$$ mathbf v_2 neq k mathbf v_1 $$

где k — некоторое число.

Рассмотрим пример трех векторов. Чтобы эти три вектора были линейно независимы, не должно быть возможности выразить третий вектор через линейные комбинации (сложение и умножение на скаляр) первых двух.

$$ mathbf v_3 neq k_1 mathbf v_1 + k_2 mathbf v_2 $$

Если так сделать нельзя, мы попадаем в трехмерное пространство, если можно — останемся на плоскости.

Линейная оболочка

Линейная оболочка (linear span) — это множество всех возможных линейных комбинаций с помощью данного набора векторов.

Если у нас два линейно независимых (двумерных) вектора, то оболочка — $R^2$ (плоскость), если три (трехмерных) вектора, но один из них линейно зависим, то по-прежнему $R^2$.

Оболочка — это ответ на вопрос, какие векторы можно построить с помощью сложения и умножения на скаляр $n$ n-мерных векторов. Линейно зависимый вектор находится внутри оболочки, создаваемой комбинациями других линейно независимых векторов.

Базис пространства

Имея два двумерных линейно независимых вектора (например, $begin{bmatrix} 1 \ 2 end{bmatrix}$ и $begin{bmatrix} 3 \ 1 end{bmatrix}$), мы можем представить любой другой вектор в пространстве $ R^2 $, сложив эти два вектора и умножив их на скаляр. Такие векторы называются базисом пространства (basis of a vector space) $ R^2 $, по сути его координатами.

В целом, базисом называется такое множество линейно независимых векторов внутри векторного пространства, с помощью которых можно выразить любой другой вектор этого пространства.

Базис можно представить как некоторую систему координат, которой пользуются все векторы данного пространства.

Повторим пример с единичными векторами $mathbf i$ и $mathbf j$.

i = np.array([1, 0])

j = np.array([0, 1])

ax = plt.axes()

plt.xlim([0.07, 3])

plt.ylim([0.07, 3])

plt.grid()

ax.arrow(0, 0, i[0], i[1], width = 0.02, head_width = 0.1, head_length = 0.2, length_includes_head = True, fc = ‘g’, ec = ‘g’)

ax.arrow(0, 0, j[0], j[1], width = 0.02, head_width = 0.1, head_length = 0.2, length_includes_head = True, fc = ‘g’, ec = ‘g’)

plt.show()

единичные векторы i и j

Приведенный выше базис называется стандартным (standard, natural basis). Это самый «экономный» или удобный способ представить все остальные векторы этого пространства.

При этом выбор такого базиса конечно условен, ничто не мешает мне перейти к другой системе координат, то есть другому базису.

Можно сказать, что координаты вектора имеют смысл только если мы знаем в какой системе координат (каком базисе) они выражены. При этом верно и то, что вектор существует в пространстве вне зависимости от системы координат или базиса.

Смена базиса

Предположим, что у нас есть два вектора исходного стандартного базиса $ mathbf g_1 $ и $ mathbf g_2 $ (на рисунке ниже изображены зеленым цветом). Кроме этого, у нас есть вектор $mathbf r$ (красный). Эти векторы имеют следующие координаты

$$ mathbf g_1 = begin{bmatrix} 1 \ 0 end{bmatrix}, mathbf g_2 = begin{bmatrix} 0 \ 1 end{bmatrix}, mathbf r_g = begin{bmatrix} 3 \ 4 end{bmatrix} $$

Если векторы нового базиса ортогональны (это важно), то мы можем выразить координаты вектора $mathbf r$ в новом базисе. Новым базисом будут следующие векторы $ mathbf b_1 $ и $ mathbf b_2 $ (черные):

$$ mathbf b_1 = begin{bmatrix} 2 \ 1 end{bmatrix}, mathbf b_2 = begin{bmatrix} -2 \ 4 end{bmatrix} $$

Посмотрим на эти векторы на графике.

1

2

3

4

5

6

7

8

9

10

11

12

13

14

15

16

17

18

19

20

21

g1 = np.array([1, 0])

g2 = np.array([0, 1])

r = np.array([3, 4])

b1 = np.array([2, 1])

b2 = np.array([2, 4])

ax = plt.axes()

plt.xlim([2.5, 4.5])

plt.ylim([0.07, 4.5])

plt.grid()

ax.arrow(0, 0, g1[0], g1[1], width = 0.02, head_width = 0.1, head_length = 0.2, length_includes_head = True, fc = ‘g’, ec = ‘g’)

ax.arrow(0, 0, g2[0], g2[1], width = 0.02, head_width = 0.1, head_length = 0.2, length_includes_head = True, fc = ‘g’, ec = ‘g’)

ax.arrow(0, 0, r[0], r[1], width = 0.02, head_width = 0.1, head_length = 0.2, length_includes_head = True, fc = ‘r’, ec = ‘r’)

ax.arrow(0, 0, b1[0], b1[1], width = 0.02, head_width = 0.1, head_length = 0.2, length_includes_head = True, fc = ‘k’, ec = ‘k’)

ax.arrow(0, 0, b2[0], b2[1], width = 0.02, head_width = 0.1, head_length = 0.2, length_includes_head = True, fc = ‘k’, ec = ‘k’)

plt.show()

смена базиса через проекцию

Убедимся, что векторы нового базиса $ mathbf b_1 $ и $ mathbf b_2 $ перпендикулярны (ортогональны).

Найдем скалярные и векторные проекции красного вектора $ mathbf r $ на векторы $ mathbf b_1 $ и $ mathbf b_2 $.

scalar_proj_r_on_b1 = np.dot(r, b1) / np.linalg.norm(b1)

vector_proj_r_on_b1 = scalar_proj_r_on_b1 * (b1 / np.linalg.norm(b1))

scalar_proj_r_on_b1.round(1), vector_proj_r_on_b1

scalar_proj_r_on_b2 = np.dot(r, b2) / np.linalg.norm(b2) ** 2

vector_proj_r_on_b2 = scalar_proj_r_on_b2 * b2

scalar_proj_r_on_b2.round(1), vector_proj_r_on_b2

Посмотрим на векторные проекции.

plt.figure(figsize = (6, 6))

ax = plt.axes()

plt.xlim([2, 4.5])

plt.ylim([0.07, 4.5])

plt.grid()

ax.arrow(0, 0, r[0], r[1], width = 0.02, head_width = 0.1, head_length = 0.2, length_includes_head = True, fc = ‘r’, ec = ‘r’)

ax.arrow(0, 0, vector_proj_r_on_b1[0], vector_proj_r_on_b1[1], width = 0.02, head_width = 0.1, head_length = 0.2, length_includes_head = True, fc = ‘k’, ec = ‘k’)

ax.arrow(0, 0, vector_proj_r_on_b2[0], vector_proj_r_on_b2[1], width = 0.02, head_width = 0.1, head_length = 0.2, length_includes_head = True, fc = ‘k’, ec = ‘k’)

plt.show()

векторные проекции в исходном базисе

В сумме векторные проекции должны дать вектор $ mathbf r_g $ в исходном базисе.

vector_proj_r_on_b1 + vector_proj_r_on_b2

В новом же базисе вектор $ mathbf r_b $ можно выразить, как скалярные проекции вектора $ mathbf r $ на векторы нового базиса $ mathbf b_1 $ и $ mathbf b_2 $.

np.array([scalar_proj_r_on_b1, scalar_proj_r_on_b2]).round(1)

Другими словами,

$$ mathbf r_b approx begin{bmatrix} 4,5 \ 2,2 end{bmatrix} $$

Ортонормированный базис

Если угол между векторами базиса равен 90 градусов, то такой базис называют ортогональным (orthogonal). Если одновременно это единичные (нормализованные) векторы, то такой базис называется ортонормированным (orthonormal).

Ортонормированный базис называют стандартным базисом пространства $R^n$.

$$ forall { mathbf q_1, …, mathbf q_k } in R^n $$

$$ mathbf q_i^T cdot mathbf q_j = begin{cases} 0, i not= j \ 1, i=j end{cases} $$

Хотя векторы базиса не обязательно должны быть ортогональными и иметь единичную норму, во многих случаях это удобно.

Видео про линейную оболочку⧉.

Подведем итог

Мы ввели понятие векторного пространства, подпространства, линейной комбинации векторов, понятия базиса, линейной независимости векторов и линейной оболочки.

Перейдем к изучению матриц и начнем этот путь с рассмотрения линейных преобразований.

Добавить комментарий